Anda di halaman 1dari 91

PREMIX

COMPILATION FOR
2020
Month – 1
th st
(4 June to 1 July)

1
www.YouTube.com/SleepyClases
www.SleepyClasses.com
2
www.YouTube.com/SleepyClases
www.SleepyClasses.com
INDEX
Topics Page No.

1. HISTORY - 1 4–7
2. HISTORY – 2 8 – 11
3. HISTORY – 3 12 – 14
4. HISTORY – 4 15 – 17
5. POLITY – 1 18 – 21
6. POLITY – 2 22 – 24
7. POLITY – 3 25 – 28
8. POLITY – 4 29 – 31
9. ECONOMY – 1 32 – 34
10. ECONOMY – 2 35 – 37
11. ECONOMY – 3 38 – 40
12. ECONOMY – 4 41 – 44
13. ENVIRONMENT – 1 45 – 48
14. ENVIRONMENT – 2 49 – 53
15. ENVIRONMENT – 3 54 – 59
16. ENVIRONMENT – 4 60 – 66
17. SCI & TECH – 1 67 – 69
18. INTERNATIONAL RELATION – 1 70 – 73
19. INTERNATIONAL RELATION – 2 74 – 76
20. SCI & TECH – 2 77 – 78
21. GEOGRAPHY – 1 79 – 81
22. GEOGRAPHY – 2 82 – 84
23. GEOGRAPHY – 3 85 – 88
24. GEOGRAPHY – 4 89 – 91

3
www.YouTube.com/SleepyClases
www.SleepyClasses.com
HISTORY - 1
Questions

1) Which of the following statements is/are correct about Zamindari system under the Mughals?
1. The Mughals appointed people to act as tax officers, sending them around the country to oversee
collection of revenue. These people were known as the zamindars
2. All categories of zamindars under the Mughals were required to perform certain police, judicial
and military duties.
3. Zamindars under the Mughals were, in fact, more the public functionaries than revenue
collecting agents.
4. Although zamindaris were allowed to be held hereditarily, the holders were not considered to be
the proprietors of their estates.
a) 1, 2 and 3 only
b) 2, 3 and 4 only
c) 1, 3 and 4 only
d) All of the above
2) Which of the following statements is/are correct about Charter Act 1813?
1. It ended the trade monopoly of the East India Company in India except for trade in tea and trade
with China.
2. There was a provision that Company should spend Rs 1 lakh every year on the education of
Indians.
3. It saw the first faint beginnings of Central Legislature in India.
a) 1 and 2 only
b) 2 and 3 only
c) 1 and 3 only
d) All of the above
3) Which of the following statements is/are correct about Swadeshi Movement?
1. Abanindranath Tagore painted Bharat Mata as a protest against the partition of Bengal.
2. Nandalal Bose, a student of Abanindranath Tagore, was the first recipient of a scholarship
offered by the Indian Society of Oriental Art , founded in 1907.
a) 1 only
b) 2 only
c) Both 1 and 2
d) Neither 1 nor 2
4) Which of the following organizations are correctly matched with their leaders?
1. The East India Association ------------- Dadabhai Naoroji
2. The Indian League ------------------------ Sisir Kumar Ghosh
3. The Indian Association of Calcutta --- Surendranath Bannerjee and Anand Mohan Bose
a) 1 and 2 only
b) 2 and 3 only
c) 1 and 3 only
d) All of the above
5) The following statements about Mughal Paintings are in the context of which ruler?
1. He commissioned the illustrations of several literary and religious texts and established an Art
Studio
4
www.YouTube.com/SleepyClases
www.SleepyClasses.com
2. Illustrations of Persian versions of Mahabharata (Razmanama) and Ramayana were produced in
miniature form.
3. The most important work is illustration of Hamzanama.
a) Humayun
b) Akbar
c) Jahangir
d) Shah Jahan

https://youtu.be/dcoD6BcQnv8?list=PLYSuG-KZy7JB0ntaozbKLnni_iYiKGOPk

5
www.YouTube.com/SleepyClases
www.SleepyClasses.com
Explanations

1) D
Zamindars
The Mughals appointed people to act as tax officers, sending them around the country to oversee
collection of revenue. These people were known as the zamindars (intermediaries)
All categories of zamindars under the Mughals were required to perform certain police, judicial
and military duties.
Zamindars under the Mughals were, in fact, more the public functionaries than revenue collecting
agents.
Although zamindaris were allowed to be held hereditarily, the holders were not considered to be
the proprietors of their estates.
Jagirdar
It is a title created during the times of Delhi Sultanate. Under Delhi Sultanate, the Jagirdars were
allowed to collect taxes, revenues and maintain a standing army. This way, they received benefits
and land grants from the Delhi Sultans.
The Mughals continued the Jagirdari system while making slight changes to the way they ruled.
Under Mughals, the Jagirdars were allowed to collect taxes, revenues and in turn submit them to
the Mughal coffers. However, the option of maintaining an army was removed.
2) A

3) C

6
www.YouTube.com/SleepyClases
www.SleepyClasses.com
4) D
The East India Association was organized by Dadabhai Naoroji in 1866 in London to discuss the
Indian question and influence public men in England to promote Indian welfare

The Indian League was started in 1875 by Sisir Kumar Ghosh with the object of "stimulating the
sense of nationalism amongst the people" and of encouraging political education

The Indian Association of Calcutta superseded the Indian League and was founded in 1876 by
younger nationalists of Bengal led by Surendranath Banerjee and Ananda Mohan Bose, who were
getting discontented with the conservative and pro-landlord policies of the British Indian
Association.
It was the most important of pre-Congress associations and aimed to ◦ (i) create a strong public
opinion on political questions ◦ (ii) unify Indian people on a common political programme
5) B
Akbar commissioned the illustrations of several literary and religious texts and established an Art
Studio (karkhana)
Key painters in his karkhana were Baswan and Daswant.
Illustrations of Persian versions of Mahabharata (Razmanama) and Ramayana were produced in
miniature form.
Historical works such as Akbar Nama also remained the main themes of Mughal paintings.
The most important work is illustration of Hamzanama (it narrates the legendary exploits of Amir
Hamza, an uncle of Muhammad)
Between 1560 and 1566, the Tutinama ("Tales of a Parrot"), was also illustrated

7
www.YouTube.com/SleepyClases
www.SleepyClasses.com
HISTORY – 2
Questions

1) Which of the following statements is/are correct about Hinayana Buddhism?


1. Hinayana Buddhists think that reaching Nirvana is the ultimate goal of all Buddhists and that
you should gain Nirvana for yourself.
2. It is found mostly in the North and West, covering
3. Its early works were written in Pali
a) 1 and 2 only
b) 2 and 3 only
c) 1 and 3 only
d) All of the above
2) Which of the following is/are sites belonging to the Neolithic Period :-
1. Paiyampalli
2. Chirand
3. Chopani Mando
a) 1 and 2 only
b) 2 and 3 only
c) 1 and 3 only
d) All of the above
3) We have evidences of which of the following crops from Harappan Civilization?
1. Wheat
2. Cotton
3. Rice
a) 1 and 2 only
b) 2 and 3 only
c) 1 and 3 only
d) All of the above
4) Which of the following statements is/are true?
1. Both Pallavas and Chalukyastried to establish supremacy over the land lying between the
Krishna and the Tungabhadra.
2. This doab was again the bone of contention in late medieval times between the Vijayanagar and
the Bahmani kingdoms.
a) 1 only
b) 2 only
c) Both 1 and 2
d) Neither 1 nor 2
5) Which of the following pairs of ‗Famous Place : River‘ are correctly matched?
1. Pandharpur : Chandrabhaga
2. Tiruchirapalli : Cauvery
3. Hampi : Malaprabha
a) 1 and 2 only
b) 2 and 3 only
c) 1 and 3 only
d) All of the above

8
www.YouTube.com/SleepyClases
www.SleepyClasses.com
https://youtu.be/eyHfTNL_Kic?list=PLYSuG-KZy7JB0ntaozbKLnni_iYiKGOPk

9
www.YouTube.com/SleepyClases
www.SleepyClasses.com
Explanations

1) C

Hinayana A Yana is a vehicle. A Hinayana is a lesser vehicle while a Mahayana is a Great vehicle.
Hinayana follows the original teaching of Buddha. It emphasizes individual salvation through self
discipline and meditation. Hinayana Buddhism followers think that everyone needs to gain
Nirvana on their Hinayana Buddhists think that The Buddha was a Human instead of a God
because they think The Buddha was simply a man who found a way to Nirvana. Hinayana think
that The Buddha is an ordinary person because he has many human looking like a person, being
born like a person, living like a person Hinayana think that The Buddha is an ordinary person
because he has many human looking like a person, being born like a person, living like a person
Hinayana is followed as a teaching or philosophy It is found mostly in the South and West
covering Indo Its early works were written in Pali Hinayana A Yana is a vehicle. A Hinayana is a
lesser vehicle while a Mahayana is a Great vehicle. Hinayana follows the original teaching of
Buddha. It emphasizes individual salvation through self discipline and meditation. Hinayana
Buddhism followers think that everyone needs to gain Nirvana on theirown Hinayana Buddhists
think that The Buddha was a Human instead of a God because they think The Buddha was simply
a man who found a way to Nirvana. Hinayana think that The Buddha is an ordinary person because
he has many human-like characteristics such as looking like a person, being born like a person,
living like a person Hinayana think that The Buddha is an ordinary person because he has many
human-like characteristics such as looking like a person, being born like a person, living like a
person Hinayana is followed as a teaching or philosophy It is found mostly in the South and West
covering Indo-china and Ceylon (Sri-Lanka) Its early works were written in Pali.

2) D

Important Neolithic Sites North-Western: Mehrgarh; Burzahom Bihar and mid-Gangetic Valley
region: South India: along the rivers Bhima, Krishna, Tungabhadra and Kaveri Some important
sites Sanganakallu, Brahmagiri, Maski Utnur, Nagarjunakonda, Budihal Paiyampalliin Tamil Nadu
Paiyampalliin Tamil Nadu Paiyampalli:It is known for the excavation site in the region which has
remains of Neolithic and Megalithic periods. Chirand:It has a large pre-historic mound which is
known for its continuous archaeological record from the Neolithic age.The excavations in
Chirandhave revealed stratified Neolithic, Chalcolithic, and Iron Age settlements, and transitions
in human habitation patterns dating from 2500 BCE to 30 CE. ChopaniMando:Itis an important
archaeological site, which indicates transition of humans from food gathering society to food
production society. It is situated inBelan river state, India.A three phase sequence of paleolithic,
Mesolithic and Neolithic is attributed by Important Neolithic Sites Burzahom; Gufkral; Koldihwa;
Mahagara Gangetic Valley region: Chirand South India: along the rivers Bhima, Krishna,
Tungabhadra and Kaveri Some important sites Maski, Piklihal, Hallurin Karnataka Budihalin
Andhra Pradesh in Tamil Nadu in Tamil Nadu It is known for the excavation site in the region
which has remains of Neolithic and Megalithic periods. historic mound which is known for its
continuous archaeological record from the have revealed stratified Neolithic, Chalcolithic, and Iron
Age settlements, and transitions in human habitation patterns dating from 2500 BCE to 30 CE. is
an important archaeological site, which indicates transition of humans from food gathering Belan
rivervalley in modern Allahabad district of Uttar Pradesh , Mesolithic and Neolithic is attributed
by archeologists.

3) D

10
www.YouTube.com/SleepyClases
www.SleepyClasses.com
The Indus people produced wheat, barley, rai, peas, sesamum, mustard Earliest people to produce
cotton. A piece of woven cloth As early as 1800 BCE, the people of Lothalgrew rice, the remains
of which have been found embedded in pottery The Indus people produced wheat, barley, rai, peas,
sesamum, mustard woven cloth has been found at Mohenjodaro. , the remains of which have been
found in the form of husks embedded in pottery.
4) C

the Pallavasand the Chalukyaschampioned Brahmanism, performed Vedic sacrifices, and made
grants to the brahmanas, the two quarrelledwith each other over plunder, prestige, and territorial
resources. Both tried to establish supremacy over the land lying between the Krishna and the
Tungabhadra. Time and again, the Pallavaprinces tried to cross the Tungabhadra, which formed
the natural historic boundary between many a kingdom of the Deccan and the deep south.
This doab was again the bone of contention in late medieval times between the Vijayanagar and
the Bahmani kingdoms.

5) A

rpuris a well known pilgrimage town on the banks of India. Vithoba temple attracts about a
millionHindupilgrims during the major July). It is a major holy place in Maharashtra and it is also
called South Kashi in Maharashtra. It is famous for the Lord temple situated on the bank of Bhima
river. Bhima river is also known as Chandrabhaga as it takes shape like crescent moon near the
town.
Tiruchirappalli sits almost at the geographic Tiruchirappalli sits almost at the geographic Cauvery
Deltabegins 16 kilometreswest of the city where the ofSrirangam, which is now incorporated in
Hampiis situated on the banks of theTungabhadra River border withAndhra Pradesh.
is a well known pilgrimage town on the banks ofChandrabhaga RiverinSolāpur district,
Maharashtra, India. pilgrims during the majoryātrā(pilgrimage) inAshadh July). It is a major holy
place in Maharashtra and it is also called South Kashi in Maharashtra. It is famous for the Lord
temple situated on the bank of Bhima river. Bhima river is also known as Chandrabhaga as it takes
shape like crescent moon near the town.
sits almost at the geographic centreof Tamil Nadu. sits almost at the geographic centreof Tamil
Nadu. west of the city where theKaveri riversplits into two, forming the island Srirangam, which is
now incorporated inTiruchirappalli
Tungabhadra Riverin the eastern part of central Karnataka near the state Andhra Pradesh.

11
www.YouTube.com/SleepyClases
www.SleepyClasses.com
HISTORY – 3
Questions

1) Welby Commission is famous in Modern Indian History for which of the following reasons?
a) Educational reforms
b) Relations between British crown and Indian princely states
c) Police reforms
d) To enquire into the Drain theory
2) Which of the following is/are correct about Charter Act 1813?
1. It completely abolished the monopoly of East India Company in trade with India
2. It completely abolished the monopoly of East India Company in trade with China
a) 1 only
b) 2 only
c) Both 1 and 2
d) Neither 1 nor 2
3) In political matters the East India Company was subordinated to the British government directly.
By which act did this happen?
a) Regulating Act, 1773
b) Pitt‘s India Act, 1783
c) Charter Act, 1793
d) None of the above
4) Which of the following newspapers is/are correctly matched with their editors?
1. Swadesamitran : G. Subramaniya Iyer
2. Bengalee : Surendranath Banerjea
3. Sudharak : NN Sen
Choose the correct option
a) 1 and 2 only
b) 2 and 3 only
c) 1 and 3 only
d) All of the above
5) Which of the following statements is/are correct?
1. Dadabhai Naoroji put forward the ‗drain of wealth‘ theory in his book Economic History in
India.
2. R. C. Dutt promoted the same theory in his book Poverty and Un-British Rule in India.
a) 1 only
b) 2 only
c) Both 1 and 2
d) Neither 1 nor 2

https://youtu.be/Q1nOq-2xMv8?list=PLYSuG-KZy7JB0ntaozbKLnni_iYiKGOPk

12
www.YouTube.com/SleepyClases
www.SleepyClasses.com
Explanations

1) D
Dadabhai Naoroji in his famous book Poverty and UnBritish Rule in India wrote his Drain Theory.
He showed how India‘s wealth was going away to England in the form of: (a) salaries,(b)savings,
(c) pensions, (d) payments to British troops in India and (e) profits of the British companies.
In fact, the British Government was forced to appoint the Welby Commission, with Dadabhai as
the first Indian as its member, to enquire into the matter.
2) D
Charter Act 1813
The Charter Act of 1813 In England, the business interests were pressing for an end to the
Company's monopoly over trade in India because of a spirit of laissez faire and the continental
system by Napoleon by which the European ports were closed to Britain.
The 1813 Act sought to redress these grievances
• The Company's monopoly over trade in India ended, but the Company retained the trade with
China and the trade in tea.
• The Company was to retain the possession of territories and the revenue for 20 years more,
without prejudice to the sovereignty of the Crown. (Thus, the constitutional position of the British
territories in India was defined explicitly for the first time.)
• Powers of the Board of Control were further enlarged.
• A sum of one lakh rupees was to be set aside for the revival, promotion and encouragement of
literature, learning and science among the natives of India, every year. (This was an important
statement from the point of state's responsibility for education.)
3) B
East India Company Act 1773 (Regulating Act, 1773)
By the Regulating Act of 1773 (later known as the East India Company Act 1773), the Parliament
of Great Britain imposed a series of administrative and economic reforms; this clearly established
Parliament's sovereignty and ultimate control over the company.
The Act recognized the company's political functions and clearly established that the "acquisition
of sovereignty by the subjects of the Crown is on behalf of the Crown and not in its own right".
East India Company Act 1784 (Pitt's India Act)
The bill differentiated the East India Company's political functions from its commercial activities.
In political matters, the East India Company was subordinated to the British government directly
To accomplish this, the Act created a Board of Commissioners for the Affairs of India, usually
referred to as the Board of Control.
4) A
1 and 2 are correctly matched. 3 is incorrect.
Sudharak : Gopal Krishna Gokhale
Indian Mirror : N.N. Sen.
5) D
The Drain of Wealth theory was systemically initiated by Dadabhai Naoroji in 1867 and further
analysed and developed by R.P. Dutt, M.G Ranade etc

13
www.YouTube.com/SleepyClases
www.SleepyClasses.com
In 1867, Dadabhai Naoroji put forward the ‗drain of wealth‘ theory in which he stated that the
Britain was completely draining India. He mentioned this theory in his book Poverty and Un-
British Rule in India.
Further in his book , he stated the loss of 200-300 million pounds of revenue to Britain.
On the footsteps of Dadabhai Naoroji, R. C. Dutt also promoted the same theory by keeping it as a
major theme of his book Economic History in India.
Dadabhai Naoroji gave several factors that caused external drain.
These are:
 Home charges refer to the interest on public debt raised in England at comparatively higher
rates; expenditure incurred in England by the Secretary of State on behalf of India;
 Annuities on account of railway and irrigation works;
 Indian office expenses including pensions to retired officials who had worked in India or
England, pensions to army and navals etc.
 Remittances to England by Europeans to their families
 Remittances for purchase of British Goods for consumption of British employees in India.
Interest charges on public debt held in Britain
 Trade as well as Indian labour was deeply undervalued.

14
www.YouTube.com/SleepyClases
www.SleepyClasses.com
HISTORY - 4
Questions

1) Which of the following statement(s) are true about animals in Harappan Civilization?
1. Animal bones found at Harappan sites include those of cattle, sheep, goat, buffalo and pig.
2. Studies done by archaeo-zoologists or zooarchaeologists indicate that these animals were
domesticated.
3. Bones of wild species such as boar, deer and gharial are also found.
a) 1 and 2 only
b) 2 and 3 only
c) 1 and 3 only
d) All of the above
2) Which of the following statement(s) are true about water storage and usage in Harappan
civilization?
1. Most Harappan sites required irrigation for agriculture
2. Traces of canals have been found
3. Water reservoirs have been found at Kalibangan
a) 1 and 2 only
b) 2 and 3 only
c) 1 and 3 only
d) All of the above
3) Which of the following statement(s) are correct about Citadel and Lower Town?
1. The Citadel owes its height to the fact that buildings were constructed on mud brick platforms.
2. It was walled, which meant that it was physically separated from the Lower Town.
3. The Lower Town was not walled
a) 1 and 2 only
b) 2 and 3 only
c) 1 and 3 only
d) All of the above
4) Arrange the following in correct chronological order.
1. Invasion of Alexander of Macedon.
2. Accession of Chandragupta Maurya
3. Accession of Nanda dynasty
Choose the correct option:-
a) 3-2-1
b) 1-2-3
c) 2-1-3
d) 3-1-2
5) Answer the following about Megasthenes
1. He was an ambassador in the court of Asoka
2. He mentions the Pandyas and says that their kingdom was famous for pearls
a) 1 only
b) 2 only
c) Both 1 and 2
d) Neither 1 nor 2

15
www.YouTube.com/SleepyClases
www.SleepyClasses.com
https://youtu.be/NjvTSlm_1yE?list=PLYSuG-KZy7JB0ntaozbKLnni_iYiKGOPk

16
www.YouTube.com/SleepyClases
www.SleepyClasses.com
Explanations

1) D
Animal bones found at Harappan sites include those of cattle, sheep, goat, buffalo and pig.
Studies done by archaeo-zoologists or zooarchaeologists indicate that these animals were
domesticated.
Bones of wild species such as boar, deer and gharial are also found. We do not know whether the
Harappans hunted these animals themselves or obtained meat from other hunting communities.
Bones of fish and fowl are also found.
NCERT Class 12 Ch 1
2) A
Most Harappan sites are located in semi-arid lands, where irrigation was probably required for
agriculture.
Traces of canals have been found at the Harappan site of Shortughai in Afghanistan, but not in
Punjab or Sind.
It is also likely that water drawn from wells was used for irrigation.
Besides, water reservoirs found in Dholavira (Gujarat) may have been used to store water for
agriculture.
Please refer NCERT Class 12 Chapter 1
3) A
The Citadel owes its height to the fact that buildings were constructed on mud brick platforms. It
was walled, which meant that it was physically separated from the Lower Town.
The Lower Town was also walled.
Several buildings were built on platforms, which served as foundations.
Please refer NCERT Class 12 Chapter 1
4) D
c. 345 BCE Origin of the Nanda dynasty
c. 327-325 BCE Invasion of Alexander of Macedon
c. 321 BCE Accession of Chandragupta Maurya

Please refer NCERT Class 12 Chapter 2


5) B
Megasthenes was an ambassador in the court of Chandragupta Maurya

17
www.YouTube.com/SleepyClases
www.SleepyClasses.com
POLITY – 1
Questions

1) Consider the following statements and mark the correct ones:


1. If a sitting Rajya Sabha member contests and wins a Lok Sabha election, his/her seat in the
Upper House becomes automatically vacant on the date he/she is declared elected to Lok Sabha 2.
If a sitting Lok Sabha member contests and wins a Rajya Sabha election, he/she has to notify the a
Rajya Sabha election, he/she has to notify the ECI in writing within 10 days the choice between
the two seats
a) 1 only
b) 2 only
c) Both 1 and 2
d) Neither 1 nor 2
2) The global report, titled Women in Business and Management: The business case for change, was
released by which of the following organisations?
a) UNDP
b) WEF
c) IMF
d) ILO
3) Which of the following is correct about the Charter Act of 1833?
1. Governor General in Council was given the power to legislate for the whole of the British
territories in India
2. The Act added law member to the Executive council of the Governor General who had no
council of the Governor General who had no vote.
a) 1 only
b) 2 only
c) Both 1 and 2
d) Neither 1 nor 2
4) Which of the following is correct about the High Courts in India?
1. The original Constitution authorised the Parliament to establish a common high court for two or
more states
2. The writ jurisdiction of High Courts is more expansive than that of Supreme Courts in India
a) 1 only
b) 2 only
c) Both 1 and 2
d) Neither 1 nor 2
5) Which of the following is correct with regard to the ‗Rights of Persons With Disabilities Act,
2016‘?
1. Attack Victims have been included for the first time within the provisions of disability
2. Every child with benchmark disability between the age group of 6 and 14 years shall have the
right to free education
3. Reservation in vacancies in government establishments has been increased by 1% for certain
persons with benchmark disability
a) 1 and 2 only
b) 2 and 3 only
18
www.YouTube.com/SleepyClases
www.SleepyClasses.com
c) 1 and 3 only
d) All of the above

https://youtu.be/ojlUr4i0WhY?list=PLYSuG-KZy7JB0ntaozbKLnni_iYiKGOPk

19
www.YouTube.com/SleepyClases
www.SleepyClasses.com
Explanations

1) A
• With regard to the Houses of Parliament, if a person is elected simultaneously to both Rajya
Sabha and Lok Sabha, and if he has not yet taken his seat in either House, he can choose, within 10
days from the later of the dates on which he is chosen to those Houses, the House of which he
would like to be a member, failing which his seat in Rajya Sabha will fall vacant at the end of this
period. [Article 101(1) of the Constitution read with Section 68(1) of The Representation of the
with Section 68(1) of The Representation of the PeopleAct, 1951]
• No such option is, however, available to a person who is already a member of one House and has
contested the election for membership of the other House. So, if sitting Rajya Sabha member
contests and wins a Lok Sabha election, his seat in the Upper House becomes automatically vacant
on the date he is declared elected to Lok Sabha to Rajya Sabha. The same applies to a Lok Sabha
member who contests an election.
2) D
• The International Labour Organization (ILO) recently released its second global report, titled
Women in Business and Management: The business case for change. Its key findings included:
• A critical mass of 30 per cent women is need by the enterprises in order to reap the benefits of
gender diversity.
• Almost half of the surveyed enterprises reported women holding less than 30 per cent of entry-
level management positions.
• In 60 per cent of companies, fewer than 30 per cent of senior
• In 60 per cent of companies, fewer than 30 per cent of senior managers and top executives are
women.
• Across the world, men are still more likely to participate in the labour market than women. The
average global labour force participation rate of women in 2018 stood at 48.5 per cent, while that
of men was 75 per cent. This equates to a 26.5 percentage point gender gap in labour force
participation.
• Just for Asia and the Pacific, the average female labour force participation rate has declined from
52.9 in 1991 to 45.3 per cent in 2018, dropping by 7.6 percentage points.
3) C
• By the Act of 1833, the Governor General in Council were given the power to legislate for the
whole of the British territories in India. These laws were applicable to all persons, British or lndian
foreigners or others and to the servants of the Company.
• They were enforceable by all courts in India.
• They were enforceable by all courts in India. The Act added one more member to the Executive
council of the Governor General, the Law Member, whose work was fully legislative. He had no
vote in the Council and he was to attend meetings, on invitation. But he practically became a
regular member of the council. Lord Macaulay, the Law member, influenced the educational
policy of the government for a number of years.
4) B
• The Constitution of India provides for a high court for each state, but the Seventh Amendment
Act of 1956 authorised the Parliament to establish a common high court for two or more states or

20
www.YouTube.com/SleepyClases
www.SleepyClasses.com
for two or more states and a union territory. The territorial jurisdiction of a high court is co-
terminus with the territory of a state.
• The writ jurisdiction of High Courts is co-terminus with the Supreme Court and in fact extends
beyond with the Supreme Court and in fact extends beyond fundamental rights to include ordinary
legal right as well. The 42nd amendment tried to erode the same but was restored by the 44th
amendment. In the Chandra Kumar case (1997), High Court‘s this power was included in the Basic
structure.
5) C
• Disability has been defined based on an evolving and dynamic concept. The types of disabilities
have been increased from existing 7 to 21 and the Central Government will have the power to add
more types of disabilities.
• Speech and Language Disability and Specific Learning Disability have been added for the first
time. Acid Attack Victims have been included. Dwarfism, muscular dystrophy have has been
indicated as separate class of specified disability. The New categories of disabilities also specified
disability. The New categories of disabilities also included three blood disorders, Thalassemia,
Hemophilia and Sickle Cell disease.
• Reservation in vacancies in government establishments has been increased from 3% to 4% for
certain persons or class of persons with benchmark disability. Every child with benchmark
disability between the age group of 6 and 18 years shall have the right to free education.

21
www.YouTube.com/SleepyClases
www.SleepyClasses.com
POLITY – 2
Questions

1) Which of the following is true in context of the Sexual Harassment at Workplace Act, 2013?
1. It covers only organised, both private and public though
2. It includes all women regardless of position and their age
a) 1 only
b) 2 only
c) Both 1 and 2
d) Neither 1 nor 2
2) Which of the following is responsible for the creation of ministries/ departments in India?
a) Prime Minister‘s Office
b) Cabinet Committee on Parliamentary Affairs
c) President
d) Parliament
3) The strength of a High Court is determined by –
a) Chief Justice of the High Court
b) Chief Justice of the Supreme Court
c) President of India
d) Parliament of India
4) Which of the following is/are true with respect to the State of Jammu and Kashmir under the
provisions given in Article 370?
1. Article 1 is not applicable to the state of Jammu and Kashmir
2. The Union of India has no power to declare Financial Emergency under Article 370 in the state
3. Part IV (Directive Principles of the State Policy) and Part IVA (Fundamental Duties) of the
Constitution are not applicable to Jammu and Kashmir.
4. In addition to other fundamental rights, Articles 19(1)(f) and 31(2) of the Constitution are still
applicable to Jammu and Kashmir
a) 2 only
b) 1,3 and 4 only
c) 2,3 and 4 only
d) Neither 1 nor 2
5) The Union Executive consists of which of the following–
1. President
2. Vice President
3. CAG
4. Attorney General of India
5. Council of Ministers
a) 1,2 and 5 only
b) 1,2,4 and 5 only
c) 1 and 5 only
d) All of the above

22
www.YouTube.com/SleepyClases
www.SleepyClasses.com
https://youtu.be/SS3Go5TFkNA?list=PLYSuG-KZy7JB0ntaozbKLnni_iYiKGOPk

Explanations

1) B
• As per the Sexual Harassment at Workplace act, the definition of "aggrieved woman", who will
get protection under the Act is extremely wide to cover all women, irrespective of her age or
employment status, whether in the organised or unorganised sectors, public or private and covers
clients, customers and domestic workers as well.
• An employer has been defined as any person who is responsible for management, supervision,
and control of the workplace and includes persons who formulate and administer policies of such
an organisation under Section 2(g).
• While the "workplace" in the Vishaka Guidelines is confined to the traditional office set-up
where there is a clear employer-employee relationship, the Act goes much further to include
organisations, department, office, branch unit etc. in the public and private sector, organized and
unorganized, hospitals, nursing homes, educational institutions, sports institutes, stadiums, sports
complex and any place visited by the employee during the course of employment including the
transportation. Even non-traditional workplaces which involve tele-commuting will get covered
under this law.
2) C
The Government of India (Allocation of Business) Rules, 1961 is made by the President of India
under Article 77 of the Constitution for the allocation of business of the Government of India.

The Ministries/Departments of the Government of India are created by the President on the advice
of the Prime Minister under these Rules. The business of the Government of India are transacted in
the ministries/departments, secretariats and offices (referred to as "Department") as per the
distribution of subjects specified in these Rules. Each of the Ministry (ies) will be assigned to a
Minister by the President on the advice of the Prime Minister. Each department will be generally
under the charge of a Secretary to assist the Minister on policy matters and general administration.
3) C
The Constitution does not specify the strength of a high court and leaves it to the discretion of the
president. Accordingly, the President determines the strength of a High Court from time to time
depending upon its workload. Every High Court (whether exclusive or common) consists of a chief
justice and such other judges as the president may from time to time deem necessary to appoint.
4) C
• Matters related to Defense, Foreign relations, Communication and Finance of Jammu and
Kashmir is under jurisdiction of Constitution of India. Union Legislature has very limited
jurisdiction in case of Jammu and Kashmir in comparison with other states. • Jurisdiction of
Parliament - Under Part XXI of the Constitution of India, which deals with "Temporary,
Transitional and Special provisions", the State of Jammu and Kashmir has been accorded special
status under Article 370. Even though included in 1st Schedule as 15th state, all the provisions of
the Constitution which are applicable to other states are not applicable to Jammu and Kashmir.
Government of India can declare emergency in Jammu and Kashmir and impose Governor's rule
under certain conditions. • Emergency Provisions- The Union of India has no power to declare
23
www.YouTube.com/SleepyClases
www.SleepyClasses.com
Financial Emergency under Article 370 in the state. The Union can declare emergency in the state
only in case of War or External Aggression.

• No proclamation of emergency made on the grounds of internal disturbance or imminent danger


thereof shall have effect in relation to the state unless (a) it is made at the request or with the
concurrence of the government of the state; or (b) where it has not been so made, it is applied
subsequently by the President to that state at the request or with the concurrence of the government
of that state. In December 1964, Articles 356 and 357 were extended to the state. • Fundamental
Duties, Directive Principles and Fundamental Rights- Part IV (Directive Principles of the State
Policy) and Part IVA (Fundamental Duties) of the Constitution are not applicable to Jammu and
Kashmir. • In addition to other fundamental rights, Articles 19(1)(f) and 31(2) of the Constitution
are still applicable to Jammu and Kashmir; hence the Fundamental Right to property is still
guaranteed in this state. It is the only state which does not have to give a detailed record on the
money flowing in the state and where it is used and how. • In the Indian Constitutional history
only one Fundamental Right has been added so far and that is Right to Education. This right too
has not been extended to Jammu and Kashmir.
5) B
•Articles 52 to 78 in Part V of the Constitution deal with the Union executive.
•The Union executive consists of the President, the Vice-President, the Prime Minister, the council
of ministers and the attorney general of India. •Please note that there is no CAG here.

24
www.YouTube.com/SleepyClases
www.SleepyClasses.com
POLITY – 3
Questions

1) Which of the following statements are correct?


1. There is a Panel of 10 Chairpersons in case the office of both Speaker and Deputy Speaker falls
vacant in LokSabha
2. The Speaker resigns from his Office by submitting his resignation to the Deputy submitting his
resignation to the Deputy Speaker.
a) 1 only
b) 2 only
c) Both 1 and 2
d) Neither 1 nor 2
2) Which of the following is correct about clause 6 of the Assam Accord?
a) Promote speedy all round economic development of Assam, so as to improve the standard of
living of the people
b) Government will arrange for the issue of citizenship certificates in future only by the
authorities of the Central Government
c) Safeguards shall be provided to protect, preserve and promote the cultural, social, linguistic
identity and heritage of the Assamese people
d) All persons who came to Assam prior to 1.1.1966, including those amongst them whose name
appeared on the electoral rolls used in 1967 elections, shall be regularized
3) Which of the following are to be considered as amendments of the Constitution under Article 368?
1. Laws made for admission or establishment of new states
2. Cessation of Indian territory to a foreign state
3. Settlement of boundary dispute between India and another country
4. Change in representation of states in the RajyaSabha
a) 1 and 2 only
b) 2 and 3 only
c) 2 and 4 only
d) 2,3 and 4 only
4) Which of the following statements are correct about Metropolitan areas?
1. Metropolitan areas have a population of ten lakhsor more
2. They comprise of one or more districts, consisting of two or more municipalities but no
Panchayats
a) 1 only
b) 2 only
c) Both 1 and 2
d) Neither 1 nor 2
5) Which of the statements are correct about elections to Rajya Sabha?
1. Elections to RajyaSabhaare held through proportional representation with single transferable
vote
2. When casual vacancies, each vacancy from a state is filled by a separate election
a) 1 only
b) 2 only
c) Both 1 and 2
25
www.YouTube.com/SleepyClases
www.SleepyClasses.com
d) Neither 1 nor 2

https://youtu.be/ANJlWI2XclA?list=PLYSuG-KZy7JB0ntaozbKLnni_iYiKGOPk

26
www.YouTube.com/SleepyClases
www.SleepyClasses.com
Explanations

1) A
When the Offices of both the Speaker and the Deputy Speaker fall vacant, the duties of the Office
of the Speaker are performed bysuchMember of the Lok Sabhaas the President may appoint for the
purpose. The person so appointed is the purpose. The person so appointed is known as the Speaker
pro tem. In case of resignation, Speaker submits his resignation to Deputy Speaker while the
Deputy Speaker submits it to the Speaker.
2) C
Clause 6 of the Assam Accord provides for constitutional, legislative and administrative
safeguards, as may be appropriate, shall be provided to protect, preserve and promote the cultural,
social, linguistic identity and heritage of the Assamese people. The Union Cabinet in January
approved the setting up of a High Level Committee for implementation of Clause 6 of the Assam
Accord and measures envisaged in the Memorandum of Settlement, 2003 and other issues related
to Bodocommunity.
The Committee shall examine the effectiveness of actions since 1985 to implement Clause 6 of the
Assam Accord. The Committee 1985 to implement Clause 6 of the Assam Accord. The Committee
will holddiscussions with all stakeholders and assess the required quantum of reservation of seats
in Assam Legislative Assembly and local bodies for Assamese people. The Committee will also
assess the requirement of measures to be taken to protect Assamese and other indigenous
languages of Assam, quantum of reservation in employment under Government of Assam and
other measures to protect, preserve and promote cultural, social, linguistic identity and heritage of
Assamese people.
3) C
Constitution (Article 4) itself declares that laws made for admission or establishment of new states
(under Article 2) and formation of new states and alteration of areas, boundaries or names of
existing states (under Articles 3) are not to be considered as amendments of the Constitution under
Article 368. Change in representation of seats of states in Rajya Sabharequires both an amendment
and ratification by half the states as provided under Article 368. The Supreme Court in the
BerubariUnion case held that the power of Parliament to diminish the area of a state (under Article
3) does not cover Parliament to diminish the area of a state (under Article 3) does not cover
cession of Indian territory to a foreign country. Hence, Indian territory can be ceded to a foreign
state only by amending the Constitution under Article 368. However, the Supreme Court in 1969
ruled that, settlement of a boundary dispute between India and another country does not require a
constitutional amendment. It can be done by executive action as it does not involve cession of
Indian territory to a foreign country.
4) C
• Indian census considers an area as urban only if it fulfillsthefollowingcriteria:
(a) The population of the settlement should be 5000 or more
(b) Densityofatleast400personspersq.km
(c) Atleast 75% of the male workers engaged in non agriculturaloccupations
• By the 74th Amendment Act, 1992, Article
• By the 74th Amendment Act, 1992, Article 243P(c) of the Constitution defines ‗metropolitan
areas‘ as those having ―population of ten lakhs [a million] or more, comprised in one or more
27
www.YouTube.com/SleepyClases
www.SleepyClasses.com
districts and consisting of two or more municipalities/panchayats/ other contiguous areas, specified
by the governor through public notification to be ametropolitanarea‖.
5) C
• Indirect elections are held to fill seats in RajyaSabha through proportional representation via
Single Transferable Vote. A single election for a number of seats in a state is held when these MPs
have completed their RajyaSabhaterms. When a seat falls vacant in circumstances such as now
(casual vacancies), each vacancy is filled by a separate election.
• In its press note announcing the separate vacancies, the Election Commission said this is ―in
conformity with provisions of Sections 147 to 151 of the Representation of the People Act, 1951
and has been Representation of the People Act, 1951 and has been consistent practice of the
Commission in such cases.‖ It also referred to two Delhi High Court rulings in favour of separate
elections, in 1994 and 2009.
• The cases cited by the EC were A K WaliavsUnion of India & Others, andSatyaPal
MalikvsEC.The High Court upheld the EC decision in both. In the former case in 1994, the court
dismissed a petition by a Congress MLA from Delhi who had argued that if one election was held
for three seats, the possibility could not be ruled out that the result could be different from the
outcome from three different elections.

28
www.YouTube.com/SleepyClases
www.SleepyClasses.com
POLITY – 4
Questions

1) Consider the following statements with regard to legislators:


Assertion: Legislators are not full time employees and can be practicing advocates
Reason: Legislators are not deemed to be public servants
a) If both statements are correct and the reasoning is the reason for the assertion
b) If both statements are correct but the reasoning is not the reason for the assertion
c) If the assertion statement is correct but the reasoning statement is not
d) If the assertion statement is incorrect but the reasoning statement is correct
2) What do you understand by the concept of ‗data localisation‘?
a) It requires the data to be completely collected from the country concerned and put to use for
country‘s welfare.
b) It requires data about a nation‘s' citizens or residents be collected, processed, and/or stored
inside the country, often before being transferred internationally.
c) It requires international data to be deposited in the country that the MNCs are entering for
business.
d) It requires data collected internationally to be regionally distributed as per the needs of the
local area and population.
3) The Umesh Sinha Committee was constituted for what purpose?
a) To study the use and feasibility of lateral entry in civil services
b) Suggest changes to Representation of the People Act, 1951 (RP Act) in view of social media
expansion
c) Suggest further reforms in the structure of GST and distribution of funds
d) Review AFSPA
4) What was the main issue of concern for the T.K. Vishwanathan Committee?
a) To deal with cybercrimes especially online hate speech
b) To suggest measures to strengthen banks and reduce NPAs
c) To deal with electoral reforms
d) To suggest measures to improve quality of primary education
5) Which of the following committees does not concern itself with the issue of lateral entry?
a) Alagh Committee
b) Hota Committee
c) Surendranath Committee
d) Jeevan Reddy Committee

https://youtu.be/O7vW8XkZsUg?list=PLYSuG-KZy7JB0ntaozbKLnni_iYiKGOPk

29
www.YouTube.com/SleepyClases
www.SleepyClasses.com
Explanations

1) C
Stating that legislators are not ―full time salaried employees‖, the Supreme Court ruled that
advocates who become lawmakers can continue their legal practice as there are no such restrictions
under the Advocates Act, 1961. The legislators are deemed to be public servants, but their status is
―sui generis‖ and certainly not one of a full time salaried employee of any person, government,
firm, corporation or concern as such, said the court. It ruled that legislators, being elected people‘s
representatives, occupy a seat in Parliament/ Legislative Assembly/ Legislative Council as
members, but are not full time salaried employees as such.
2) B
Data localization or data residency law requires data about a nations' citizens or residents be
collected, processed, and/or stored inside the country, often before being transferred
internationally, and usually transferred only after meeting local privacy or data protection laws,
such as giving the user notice of how the information will be used and obtaining their consent. The
new law in India requires payments companies to store all information about transactions
involving Indians solely on computers in the country. The law and the hubbub over it are part of a
debate over a concept known as ―data localization,‖ in which a country places restrictions on data
as a way to gain better control over it and potentially curb the power of international companies. In
India the B.N. Srikrishna committee recommended the same which is the basis for India‘s work in
progress data protection Bill. The Reserve Bank of India on 26th June 2019 said all data related to
payments must be stored only in India and data processed abroad will have to be brought back to
the country within 24 hours.
3) B
The Election Commission of India on 11 January 2018 constituted a 14 member committee to
suggest changes to Section 126 of the Representation of the People Act, 1951 (RP Act) in view of
social media expansion chaired by Senior Deputy Election Commissioner Umesh Sinha. The
committee will comprise of nine officers of Election Commission, one nominated member each
from the Ministry of Information and Broadcasting (I&B), Law Ministry, IT Ministry, National
Broadcasters Association and Press Council of India. • It will study the impact of new media and
social media during the ―silence period‖, the 48 hours period prior to elections and its implication
in view of Section 126 of the RP Act. • After a thorough study, it will suggest modifications to the
election law and model code of conduct. • It will examine the difficulties faced in regulating media
platforms during the prohibitory 48 hours in a multi-phase election.

4) A
An expert committee headed by former Lok Sabha secretary general TK Viswanathan to deal with
cybercrimes especially online hate speech recommended appointing cybercrime coordinators in all
states and establishing cybercrime cells in each district. The committee was formed after Supreme
Court struck down Section 66 A of the Information Technology (IT) Act, 2000.
5) D

Jeevan Reddy Committee Report deals with Armed Forces (Special Powers) Acts (AFSPA). The
committee had recommended repealing the AFSPA. All the other committees are concerned with
the issue of lateral entry. After inducting 9 private sector specialists as joint secretaries, the

30
www.YouTube.com/SleepyClases
www.SleepyClasses.com
Narendra Modi government is mulling to rope in subject experts from the private sector at the
Deputy Secretary (DS) and Director levels as well. The move is aimed at boosting efficiency and
bring more expertise into various government departments. Officials in Department of Personnel
& Training have been asked to prepare a proposal for inducting 400 domain experts to fill
DS/Director posts. Government has been rooting for lateral entry to bridge the acute talent shortfall
at the top of policy making. Officials at DoPT have also been asked to frame a process for
recruitment and evaluation of private sector employees into central administration.

31
www.YouTube.com/SleepyClases
www.SleepyClasses.com
ECONOMY – 1
Questions

1) Which of the following is/are most likely the impact(s) of a rate cut by RBI:
1. Increase in GDP Growth Rate
2. Increase in Inflation
3. Increase in Credit to Deposit Ratio
a) 1 and 2 only
b) 2 and 3 only
c) 1 and 3 only
d) All of the above
2) Which of the following correctly explains ‗on tap‘ licencing, as used by RBI:
1. Eligible entities need to wait for RBI to start the licencing process
2. It is an on-supply basis licencing process
a) 1 only
b) 2 only
c) Both are correct
d) None is correct
3) Which of the following is/are true about G20:
1. Apart from European Union, ASEAN is the only other grouping present as a member
2. While India is a member, China is not
a) 1 only
b) 2 only
c) Both are correct
d) None is correct
4) Which of the following is/are true about competition commission of india (cci):
1. It ensures that monopolies are not created in the country
2. It regulates combinations such as acquisition, acquiring of control and Mergers
a) 1 only
b) 2 only
c) Both are correct
d) None is correct
5) Which of the following is/are correct in context of rtgs (real time gross settlement):
1. The minimum amount to be remitted through RTGS (Real Time Gross Settlement) is ₹2,00,000/
2. RTGS is not a 24x7 system.
a) 1 only
b) 2 only
c) Both are correct
d) None is correct

https://youtu.be/tP6nF1-yFjs?list=PLYSuG-KZy7JB0ntaozbKLnni_iYiKGOPk

32
www.YouTube.com/SleepyClases
www.SleepyClasses.com
Explanations

1) A
IMPACT OF A RATE CUT
It reduces the interest rates, leading to a lesser amount of interest, bringing down the overall cost of
the loan. However, this will come into effect only if banks decide to pass on the benefit to their
customers.
The reduction in the repo rate means that industries may be able to get loans at cheaper interest
rates from lenders. It can result in commodities becoming cheaper due to lower interest costs. It
also strengthens domestic growth impulses by spurring private investment. A rate cut can increase
liquidity in the economy and hence drive up the cost of goods and services, leading to increase in
inflation. However, Deposit Ratio is not impacted by Repo Rate as deposits are governed by Cash
Reserve Ratio (CRR) and other factors.
2) D
‗ON TAP‘ LICENSING
It enables entities to approach the RBI for obtaining licences for starting banks or small finance
banks on meeting laid-down criteria. Eligible entities would not have to wait for licences as it
would be available ondemand basis.
3) D
THE G20 (OR GROUP OF TWENTY)
It is an international forum for the governments and central bank governors from 19 countries and
the European Union. The EU is represented by the European Commission and by the European
Central Bank. Membership includes:

4) B
COMPETITION COMMISSION OF INDIA (CCI):
A fair competition in market is essential to achieve this objective. CCI‘s goal is to create and
sustain fair competition in the economy that will provide a ‗level playing field‘ to the producers
and make the markets work for the welfare of the consumers. The Competition Act, 2002, as
amended by the Competition (Amendment) Act, 2007, prohibits anticompetitive agreements, abuse
of dominant position by enterprises and regulates combinations (acquisition, acquiring of control
and M&A), which causes or likely to cause an appreciable adverse effect on competition within
India. The objectives of the Act are sought to be achieved through the Competition Commission of
India, which has been established by the Central Government with effect from 14th October 2003.
CCI consists of a Chairperson and 6 Members appointed by the Central Government.
5) C
RTGS
RTGS stands for Real Time Gross Settlement. It is a system where there is continuous and real-
time settlement of fund-transfers, individually on a transaction by transaction basis (without

33
www.YouTube.com/SleepyClases
www.SleepyClasses.com
netting). 'Real Time' means the processing of instructions at the time they are received; 'Gross
Settlement' means that the settlement of funds transfer instructions occurs individually. NEFT is an
electronic fund transfer system in which the transactions received up to a particular time are
processed in batches. Contrary to this, in RTGS, the transactions are processed continuously on a
transaction by transaction basis throughout the RTGS business hours. RTGS is not a 24x7 system.
The RTGS service window for customer transactions is available to banks from 8 am to 4.30 pm
on a working day, for settlement at the RBI end. However, the timings that the banks follow may
vary from bank to bank. The RTGS system is primarily meant for large value transactions. The
minimum amount to be remitted through RTGS is ₹2,00,000/with no upper or maximum ceiling.

KEEP WORKING HARD


I wish I could know the secret of cracking the exam.
Working Hard is the only thing I know, It is the only thing I want from aspirants and It is the only
advice I can give as a teacher.

34
www.YouTube.com/SleepyClases
www.SleepyClasses.com
ECONOMY – 2
Questions

1) Which of the following is/are correct about NITI AAYOG:-


1. It was formed via an act of Parliament on January 1, 2015
2. The Team India Hub in NITI leads the engagement of states with the Central government.
3. The Knowledge and Innovation Hub in NITI builds NITI‘s think-tank capabilities.
a) 1 and 2 only
b) 2 and 3 only
c) 1 and 3 only
d) None of the above
2) Cabinet recently approved the new delhi international arbitration centre bill, 2019. In this regard,
consider the following statements:
1. It will promote 'Ease of Doing Business' in India
2. It is being proposed based on the recommendations of a High-Level Committee (HLC), headed
by Mr. Justice B.N. Srikrishna
a) 1 only
b) 2 only
c) Both are correct
d) None is correct
3) Base erosion and profit shifting relates with:
1. Tax holidays given by governments to promote business activity in remote or relatively under-
developed places
2. It usually results in little or no tax being paid
a) 1 only
b) 2 only
c) Both are correct
d) None is correct
4) Which of the following is/are true about economic census:
1. It is the complete count of all establishment located within the geographical boundary of India.
2. It is conducted by Ministry of Finance.
a) 1 only
b) 2 only
c) Both are correct
d) None is correct
5) In news, double deflation is used in context of:
a) Calculating the trend of price rise in an Economy
b) Estimating real value added of an industry
c) Calculating the trend of exports/imports in an Economy
d) Estimating the optimum amount of Foreign Exchange required by a country

https://youtu.be/pP6zkR2VT9I?list=PLYSuG-KZy7JB0ntaozbKLnni_iYiKGOPk

35
www.YouTube.com/SleepyClases
www.SleepyClasses.com
Explanations

1) B
NITIAAYOG
The National Institution for Transforming India, also called NITI Aayog, was formed via a
resolution of the Union Cabinet on January 1, 2015. NITI Aayog provides relevant technical
advice to the Centre and States. The Government of India, in keeping with its reform agenda,
constituted the NITI Aayog to replace the Planning Commission instituted in 1950. NITI Aayog
attempts to foster Cooperative Federalism. At the core of NITI Aayog‘s creation are two hubs –
Team India Hub and the Knowledge and Innovation Hub. The Team India Hub leads the
engagement of states with the Central government. While the Knowledge and Innovation Hub
builds NITI‘s think-tank capabilities. These hubs reflect the two key tasks of the Aayog. PM is the
Chairperson of NITI.
2) C
NEW DELHI INTERNATIONAL ARBITRATION CENTRE (NDIAC) In view of the provisions
of the Article 107 (5) and 123 (2) of the Constitution, the New Delhi International Arbitration
Centre Bill, 2019 is proposed to be introduced in the Parliament which will replace the New Delhi
International Arbitration Centre Ordinance, 2019. It will resolve International and domestic
commercial disputes expeditiously by Alternative Dispute Resolution (ADR) mechanism. A High-
Level Committee (HLC), headed by Mr. Justice B.N. Srikrishna was constituted in the year 2017.
The HLC recommended that the Government may take over the International Centre For
Alternative Dispute Resolution (ICADR), an existing institution which has been established in the
year 1995 using the public funds and develop it as an Institution of National Importance. Taking
into consideration the HLC's recommendations, a Bill, namely the New Delhi International
Arbitration Centre (NDIAC) Bill 2018 was approved for introduction in the Parliament by the
Cabinet in December, 2017.

NEW DELHI INTERNATIONAL ARBITRATION CENTRE (NDIAC) A hub of institutionalized


arbitration and promote 'Ease of Doing Business' in India. In view of the provisions of the Article
107 (5) and 123 (2) of the Constitution, the New Delhi International Arbitration Centre Bill, 2019
is proposed to be introduced in the Parliament which will replace the New Delhi International
Arbitration Centre Ordinance, 2019. The New Delhi International Arbitration Centre (NDIAC)
will be headed by a Chairperson, who has been a Judge of the Supreme Court or a Judge of a High
Court or an eminent person, having special knowledge and experience in the conduct or
administration of arbitration, law or management, to be appointed by the Central Government in
consultation with the Chief Justice of India. Besides, it will also have two Full-time or Part-time
Members from amongst eminent persons having substantial knowledge and experience in
institutional arbitration in both domestic and international. In addition, one representative of a
recognized body of commerce and industry shall be nominated on rotational basis as a Part-time
Member. The Secretary, Department of Legal Affairs, Ministry of Law & Justice, Financial
Adviser nominated by Department of Expenditure, Ministry of Finance and Chief Executive
Officer, NDIAC will be ex-officio Members.
3) B
BEPS (BASE EROSION AND PROFIT SHIFTING)
36
www.YouTube.com/SleepyClases
www.SleepyClasses.com
It refers to tax planning strategies that exploit gaps and mismatches in tax rules to artificially shift
profits to low or no-tax locations where there is little or no economic activity, resulting in little or
no tax being paid. India has ratified the Multilateral Convention to Implement Tax Treaty Related
Measures to Prevent Base Erosion and Profit Shifting, which was signed by previous Finance
Minister at Paris in June 2017. The Multilateral Convention is an outcome of the OECD / G20
Project to tackle Base Erosion and Profit Shifting, also known as the "BEPS Project―.
4) A
ABOUT ECONOMIC CENSUS
Economic Census is the complete count of all establishment located within the geographical
boundary of India. The Economic Census provides disaggregated information on various
operational and structural variables of all establishments of the country. Economic Census also
provides valuable insight into geographical spread/clusters of economic activities, ownership
pattern, persons engaged, etc. of all economic establishments in the country. The information
collected during Economic Census are useful for socio-economic developmental planning at state
and district levels. It is being conducted by Ministry of Statistics and Programme Implementation
(MoSPI) in 2019. Six Economic Censuses, (EC) have been conducted by the Union Ministry of
Statistics and Program Implementation till date.
5) B
DOUBLE DEFLATION
In news because of controversies surrounding GDP growth rate. Double deflation is the technique
used to estimate real value added of an industry. In the double deflation method, real value added
is measured as the difference between real gross output and real intermediate inputs. Double
deflation is the conceptually preferred method of computing real value added because it requires
fewer assumptions about the relationships among gross output and intermediate inputs. India has
subscribed to the Special Data Dissemination Standard (SDDS) of the International Monetary
Fund (IMF). The IMF had raised certain issues on the usage of double deflation in the Indian GDP
series and India had informed IMF that the existing data availability does not permit its application
in India at present. In view of divergent views, the Advisory Committee on National Accounts
Statistics (ACNAS) had not agreed to adoption of double deflation at this stage. Moreover, double
deflation is used in only a few countries that have a Producers Price Index (PPI) to deflate the
inputs.

37
www.YouTube.com/SleepyClases
www.SleepyClasses.com
ECONOMY – 3
Questions

1) Consider the following statements regarding kimberley process.


1. India is hosting this year‘s inter-sessional meeting of Kimberley Process in Delhi
2. Kimberley is a city situated in South Africa
3. Kimberley Process is open to all countries
Which of these options is/are incorrect:
a) 3 only
b) 2 and 3 only
c) 1 only
d) None of the above
2) Which of the statements is true in the context of WTO?
1. Kazakhstan is organizing the Twelfth Ministerial Conference (MC12) to be held in 2020.
2. The previous Ministerial Conference (MC11) was held in Geneva in December 2017.
a) 1 only
b) 2 only
c) Both are correct
d) None is correct
3) Which of the following statements are true about kaladan multimodal transit transport project?
1. India, Myanmar and Bangladesh are involved in this project.
2. Sea and Railways are the only modes of transport proposed under this project
a) 1 only
b) 2 only
c) Both are correct
d) None is correct
4) Which of the following statements is incorrect:
a) The APEDA was established by the Government of India under the Agricultural and Processed
Food Products Export Development Authority Act passed by the by the Parliament in December
1985.
b) APEDA has been entrusted with the responsibility to monitor import of sugar
c) APEDA comes under the ambit of Ministry of Agriculture
d) APEDA replaced the Processed Food Export Promotion Council (PFEPC)
5) Which of the following given statements is/are correct:
1. The Shanghai Five mechanism created in 1996 had China, Kazakhstan, Kyrgyzstan, Russia and
Tajikistan as its members.
2. The declaration of Shanghai Cooperation Organisation was signed in 2001 with Uzbekistan as
its member.
3. India and Pakistan became SCO‘s full members in the year 2017.
a) 2 only
b) 1 and 2 only
c) 1 and 3 only
d) All of the above
https://youtu.be/JzkAPhSfqXI?list=PLYSuG-KZy7JB0ntaozbKLnni_iYiKGOPk

38
www.YouTube.com/SleepyClases
www.SleepyClasses.com
Explanations

1) C
KIMBERLEY PROCESS
India is hosting this year‘s inter-sessional meeting of Kimberley Process in Mumbai.
We discussed this in today‘s PreCure.
2) A
ABOUT WTO
1) Kazakhstan is organizing the Twelfth Ministerial Conference (MC12) to be held in 2020.
2) The previous Ministerial Conference (MC11) was held in Buenos Aires in December 2017.
3) D
KALADAN MULTIMODAL TRANSIT TRANSPORT PROJECT
India & Myanmar are involved in this project. It involves all the modes of transport,that
is,road,rail,waterways and sea.
Discussed in today‘s PreCure.
4) C
APEDA
It comes under the ambit of Ministry of Commerce and Industry. The Agricultural and Processed
Food Products Export Development Authority (APEDA) was established by the Government of
India under the Agricultural and Processed Food Products Export Development Authority Act
passed by the Parliament in December, 1985.

APEDA is mandated with the responsibility of export promotion and development of the
following scheduled products:
• Fruits, Vegetables and their Products.
• Meat and Meat Products.
• Poultry and Poultry Products.
• Dairy Products.
• Confectionery, Biscuits and Bakery Products.
• Honey, Jaggery and Sugar Products.
• Cocoa and its products, chocolates of all kinds.
• Alcoholic and Non-Alcoholic Beverages.
• Cereal and Cereal Products.
• Groundnuts, Peanuts and Walnuts.
• Pickles, Papads and Chutneys.
• Guar Gum.
• Floriculture and Floriculture Products.
• Herbal and Medicinal Plants. In addition to this, APEDA has been entrusted with the
responsibility to monitor import of sugar.
5) D
SCO Founded in 2001 in Shanghai 8 members: China, Kazakhstan, Kyrgyzstan, Russia, Tajikistan,
Uzbekistan, India and Pakistan
Started as Shanghai Five (First Five mentioned above)
HQ : Beijing, China

39
www.YouTube.com/SleepyClases
www.SleepyClasses.com
2015 - SCO decided to admit India and Pakistan as full members They joined as full members in
2017 in Astana, Kazakhstan, June 2017
Main activities Cooperation on security, military activities, and economic and cultural cooperation,
Regional Antiterrorism Structure (RATS)
Latest Summit: Bishkek, Kyrgyzstan.

40
www.YouTube.com/SleepyClases
www.SleepyClasses.com
ECONOMY – 4

Questions

1) Consider the following straits and regions:


Straits Regions
1- Strait of Malacca South East Asia
2- Strait of Bab-El-Mandab West Africa
3- Strait of Tartary East Asia
4- Starit of Hormuz West Asia
Which of these is/are incorrectly matched?
a) 2 only
b) 3 and 4 only
c) 1, 2 and 3 only
d) All are correctly matched
2) The components used to make stainless steel include:
1- Chromium
2- Silicon
3- Carbon
4- Nickel
5- Molybdenum
6- Aluminium
Which of the following is correct?
a) 1,3 and 6 only
b) 2,3,4 and 6 only
c) 2,3 and 5 only
d) All of the above ingredients
3) Which of the following statements is correct in the context of indiaand renewable energy:
a) India plans to establish a Renewable Energy capacity of 500 GW by 2022
b) India targets to install 175 GW of Solar Energy capacity by 2022
c) India‘s renewable energy capacity target by 2022 also includes 60 GW from wind, 10 GW from
bio-power and 5 GW from small hydro-power
d) International Solar Alliance (ISA) has been initiated by G-20 Countries
4) Which of the following correctly describes the ‗section 7 of the rbi act‘:
a) It empowers the RBI to be the banker to the Central Government
b) It empowers the Government to issue directions to RBI
c) It decides the composition of central board of Reserve Bank of India
d) It gives authority to RBI to issue bank notes
5) Which of the following statements is correct in context of cpi?
1. The Housing Group has the maximum weightage in CPI-Rural
2. Food and Beverages Group has the maximum weightage in CPI-Urban and CPI-Combined
a) 1 only
b) 2 only
c) Both are correct

41
www.YouTube.com/SleepyClases
www.SleepyClasses.com
d) None is correct

https://youtu.be/8o971hMS4c0?list=PLYSuG-KZy7JB0ntaozbKLnni_iYiKGOPk

42
www.YouTube.com/SleepyClases
www.SleepyClasses.com
Explanations

1) A

2) D
3) C
India targets to install 175 GW of renewable energy capacity by 2022. The 175 GW of renewable
energy capacity target by 2022 includes 100 GW from solar, 60 GW from wind, 10 GW from bio-
power and 5 GW from small hydro-power. The International Solar Alliance is an alliance of more
than 122 countries initiated by India, most of them being sunshine countries, which lie either
completely or partly between the Tropic of Cancer and the Tropic of Capricorn.
4) B
RBI ACT AND VARIOUS SECTIONS
Section 7 - Empowers the Government to issue directions to RBI
Being banker to the Central Government - Section 21
Composition of central board of Reserve Bank of India - Section 8
Right to issue bank notes - Section 22
5) B

43
www.YouTube.com/SleepyClases
www.SleepyClasses.com
44
www.YouTube.com/SleepyClases
www.SleepyClasses.com
ENVIRONMENT – 1
Questions

1) Consider the following statements about the Cauvery river


1. It is the fourth-largest river in south India.
2. The Cauvery basin is spread through three states and one UT.
3. It originates in the Brahmagiri hills in Kodagu, in a place called Talakaveri in Tamil Nadu.
Which of the above statements is/are correct?
a) 2 and 3 only
b) 1 only
c) 1 and 3 only
d) 1 and 2 only
2) Consider the following statements about the World Environment Day :
1. It is celebrated on the 5th of June every year.
2. The host country for 2019 is India.
3. The theme of World Environment Day 2019 is Air Pollution.
4. The theme is decided by the host country.
Which of the above statements is/are correct?
a) 1,2 and 3 only
b) 1 and 3 only
c) 1,3 and 4 only
d) All of the above
3) Consider the following statements about House Sparrow:-
1. It is listed as ‗Endangered‘ in the IUCN Red List.
2. It is the state bird of Bihar and New Delhi.
Which of the above statements is/are correct?
a) 1 only
b) 2 only
c) Both of the above
d) None of the above
4) Grossly Polluting Industries (GPI) can be defined as :
a) They are the industries that discharge radioactive wastes into the rivers.
b) They are the industries that discharge contaminated medical waste into the rivers.
c) They are the industries that discharge more than 1,00,000 litres of wastewater and/or hazardous
chemicals into the rivers.
d) None of the above
5) Consider the following statements regarding UNICEF study on Swachh Bharat [Clean India]
mission :
1. The study is based on ground reports from three states, Bihar, Odisha and West Bengal.
2. The study has suggested that in terms of faecal contamination, non-ODF villages were more likely
to have their groundwater sources contaminated than ODF villages. 3. The study has been
commissioned by UNICEF and the Bill and Melinda Gates Foundation.
Which of the above statements is/are correct?
a) 1 and 3 only
b) 1 and 2 only
45
www.YouTube.com/SleepyClases
www.SleepyClasses.com
c) 2 and 3 only
d) All of the above

https://youtu.be/JVREqwVryIo?list=PLYSuG-KZy7JB0ntaozbKLnni_iYiKGOPk

46
www.YouTube.com/SleepyClases
www.SleepyClasses.com
Explanations

1) D
The Cauvery (also spelt as ‗Kaveri‘), known ‗Ponni‘ in Tamil, is the fourth-largest river in south
India. Originating in the Western Ghats at Talakaveri in Karnataka‘s Kodagu district, it passes
through Tamil Nadu. The river bisects the state into north and south and finally reaches the Bay of
Bengal at Poompuhar, also known as Kaveripoompattinam in Tamil Nadu. The Cauvery basin is
spread over 81,155 square kilometres (sq km) in the states of Karnataka (34,273 sq km), Tamil
Nadu (43,856 sq km) and Kerala (2,866 sq km) and the Union Territory of Puducherry (160 sq
km). The Cauvery‘s major tributaries, Kabini and Moyar, join it before it reaches the Stanley
Reservoir at Mettur in Tamil Nadu‘s Salem district. The river‘s total length, from source to mouth,
is 802 kilometres. From antiquity to the present era, the river has been the lifeline of the ancient
kingdoms and cities of south India. Because of the river‘s bountiful nature, the Cauvery delta was
considered to be one of the most fertile regions in India till recently.

2) B
World Environment Day is celebrated on the 5th of June every year, and is the United Nation's
principal vehicle for encouraging awareness and action for the protection of our environment. This
year, the theme is ‗Air Pollution‘. The theme is decided by the UNEP. Last year India was the host
country, this time China will do the honours. China will be a great global host of 2019‘s World
Environment Day celebrations. The country has demonstrated tremendous leadership in tackling
air pollution domestically.
3) B

47
www.YouTube.com/SleepyClases
www.SleepyClasses.com
The International Union for Conservation of Nature (IUCN) included the house sparrow (Passer
domesticus) among its list of "Endangered" species in 2002 due to its declining population. Its
existence had become so threatened in India in the last two decades that a special study was
commissioned in 2007 to find out the status of the sparrow population in Delhi. The IUCN has
included the house sparrows among its ―Least Concern‖ species during 2018. It is therefore
confirmed that the house sparrow population is rebounding. This may have happened due to the
combined effort by individuals of bird lovers, non-profits and government organisations.

For conserving the house sparrow and motivating people for the same, March 20, 2010, was
declared as ―World Sparrow Day‖. Delhi declared it the state bird and mascot on August 14, 2012.
On April 17, 2013, the Bihar cabinet declared the house sparrow as the state bird.
4) C
In India, both surface and groundwater resources are under stress. One of the reasons for this is the
substantial increase in the number of grossly polluting industries (GPI) between 2011 and 2018.
There has been a 136 per cent increase in the number of grossly polluting industries over the
period, according to the State of India‘s Environment (SoE) In Figures, 2019. Around 84 per cent
of the GPIs were found to be located in four states — Uttar Pradesh (1,079), Haryana (638),
Andhra Pradesh (193) and Gujarat (178). GPIs are industries that discharge more than 1,00,000
litres of wastewater and/or hazardous chemicals into the rivers, and include pulp and paper mills,
distilleries, sugar mills, textile units, tanneries, thermal power plants, the food, dairy and beverage
industries, chemical units, slaughterhouses, etc.

Grossly Polluting Industries (GPI) are defined as the industry which is discharging wastewater
more than 100KLD and/or hazardous chemicals used by the industry as specified under the
Schedule-I, Part-II of The Manufacture, Storage and Import of Hazardous Chemical Rules of 1989
under Environment (Protection) Act, 1986.
5) D
A UNICEF study has lauded the Narendra Modi-led government‘s Swachh Bharat initiative,
saying it had led to a reduction in groundwater contamination. The study, commissioned by
UNICEF and the Bill and Melinda Gates Foundation, aimed at assessing the environmental impact
and communication footprint of the Swachh Bharat Mission (Grameen). It is based on ground
reports from three states — Bihar, Odisha and West Bengal The study launched on World
Environment Day found that in terms of faecal contamination, non-ODF villages were on average
11.25 times more likely to have their groundwater sources contaminated (12.7 times more from
contaminants traceable to human alone), 1.13 times more likely to have their soil contaminated,
1.48 times more likely to have food contaminated and 2.68 times more likely to have household
drinking water contaminated.

48
www.YouTube.com/SleepyClases
www.SleepyClasses.com
ENVIRONMENT – 2
Questions

1) Consider the following statements about the Pichavaram mangroves :-


1. It is the second largest in the country after Sundarbans.
2. Avicennia marina and Rhizophora are the predominant flora.
3. It is separated from the Bay of Bengal by a sand bar.
Which of the above statements is/are correct?
a) 1 and 2 only
b) 2 and 3 only
c) 1 and 3 only
d) All of the above
2) Which of the following are Olive Ridley Nesting sites in Odisha ?
1. Gahirmatha marine sanctuary
2. Rushikulya coast
3. Bahuda river mouth
Which of the above statements is/are correct?
a) 1 and 2 only
b) 2 and 3 only
c) 1 and 3 only
d) All of the above
3) Consider the following statements about Acute Encephalitis Syndrome (AES) :-
1. It is caused by viruses only.
2. Japanese encephalitis virus (JEV) is the major cause of AES in India.
Which of the above statements is/are correct?
a) 1 only
b) 2 only
c) Both of the above
d) None of the above
4) Which of the following states given has declared Pakke Paga Hornbill Festival (PPHF) as a ― ―state
festival‖?
a) Kerala
b) Nagaland
c) Arunachal Pradesh
d) Manipur
5) Consider the following statements about End of Childhood Index :-
1. It was released by Save the Children, a non-profit, on May 28, 2019.
2. Countries are evaluated on eight parameters on the wellbeing of those up to 19 years of age —
child mortality, malnutrition, lack of education, child labour, early marriage, adolescent birth,
displacement by conflict and child homicide.
3. Singapore has topped the list.
Which of the above statements is/are correct?
a) 1 and 3 only
b) 1 and 2 only
c) 2 and 3 only
49
www.YouTube.com/SleepyClases
www.SleepyClasses.com
d) All of the above

https://youtu.be/ulkyjC06OSI?list=PLYSuG-KZy7JB0ntaozbKLnni_iYiKGOPk

50
www.YouTube.com/SleepyClases
www.SleepyClasses.com
Explanations

1) D
The Pichavaram mangroves are the second largest in the country after Sundarbans and are not only
valuable as they act as carbon sinks but also form an important line of protection for the coast from
natural disasters like cyclones, floods and tsunami. It is separated from the Bay of Bengal by a
sand bar. The Pichavaram mangrove, a bio shield that protected the stretch of the coast from
natural calamities such as the 2004 tsunami, is facing a threat from hydrocarbon exploration.
Avicennia marina and Rhizophora are the predominant flora and act as a buffer and prevent tidal
onslaught.

2) D
Beach at Bahuda river mouth in Odisha being developed to lure the turtles. The Odisha forest
department is all set to add another olive ridley mass nesting site to its wildlife map. It has started
preparing the beach at the Bahuda river mouth in Ganjam district to lure the endangered turtles to
come over for mass nesting next year. Around 3-km stretch of the beach from Sunapur to Anantpur
at Bahuda rookery is being developed as a possible olive ridley mass nesting site. The Bahuda
rookery is located around 20 km to the south of Rushikulya rookery coast, a major mass nesting
site of olive ridleys on the Indian coastline. Gahirmatha marine sanctuary and Rushikulya rookery
coast in Ganjam district are main Olive Ridley Nesting sites in Odisha. Of these sites, Gahirmatha
marine sanctuary is largest rookery (mass nesting site) of Olive Ridley turtles,

51
www.YouTube.com/SleepyClases
www.SleepyClasses.com
3) B
Acute Encephalitis Syndrome (AES) including Japanese Encephalitis (JE) is a group of clinically
similar neurologic manifestation caused by several different viruses, bacteria, fungus, parasites,
spirochetes, chemical/ toxins etc. The outbreak of JE usually coincides with the monsoon and post
monsoon period when the density of mosquitoes increases while encephalitis due to other viruses
specially entero-viruses occurs throughout the year as it is a water borne disease. Viruses are the
main causative agents in AES cases, although other sources such as bacteria, fungus, parasites,
spirochetes, chemicals, toxins and noninfectious agents have also been reported over the past few
decades. Japanese encephalitis virus (JEV) is the major cause of AES in India (ranging from 5%-
35%). Nipah virus, Zika virus are also found as causative agents for AES.
4) C
The government of Arunachal Pradesh on January 20 declared the Pakke Paga Hornbill Festival
(PPHF)–the state‘s only conservation festival, as a ―state festival‖. The first-ever PPHF was held
on January 16–18, 2015. The organizers had a number of objectives in mind—to recognise the
role played by the resident Nyishi tribe in conserving hornbills in the Pakke Tiger Reserve (PTR),
to devise alternative sources of income for a region that relies on hunting and logging, and to tell
the rest of the country about the wonders of the PTR and its surrounding areas. As part of the
festival, visitors are treated to cultural programmes, food stalls, plays, sightseeing, river and village
walks, bird-watching, screenings of short films as well as local tribal sports and dances. PPHF had
helped in creating awareness on nature conservation and lauded the Nyishi tribe for its part in
putting the festival on the national map.

52
www.YouTube.com/SleepyClases
www.SleepyClasses.com
5) D
India ranks 113 in 176 countries on the wellbeing of children, according to the End of Childhood
Index. The country stood at 116 among 172 countries in 2018. The index is part of Changing
Lives in Our Lifetime - Global Childhood Report, 2019 and was released by Save the Children, a
non-profit, on May 28, 2019. Countries are evaluated on eight parameters on the wellbeing of
those up to 19 years of age — child mortality, malnutrition, lack of education, child labour, early
marriage, adolescent birth, displacement by conflict and child homicide. In its neighbourhood,
India ranked fourth, behind Bhutan (98), Sri Lanka (56) and China (36). Singapore tops the list and
eight European countries figure among the top 10. South Korea stands at the 10th position and at
the bottom are African countries performing poorly on all indicators. However, India has
improved significantly on child mortality from 39 deaths per 1000 live births in 2017. This is still
way worse than the goal of 25 or fewer deaths set my Millennium Development Goals. More than
38 per cent Indian children are stunted. In China this number is 8.1 per cent, Sri Lanka (17.3 per
cent) and Bhutan (33.6 per cent). Every fifth Indian child is out of school at primary and
secondary levels and more than 11 per cent in the 5-17 age groups are child labourers. Globally,
the rate of child marriage has reduced by 25 per cent in 2019. In South Asia, India has performed
significantly well where child marriage is down 51 per cent since 2000 and 63 percent since 1990.
There are three million fewer teen births now than 2000 worldwide. India has successfully cut the
teenage birth rate by 63 per cent since 2000 and 75 per cent since 1990. This results in nearly two
million fewer teen births in the country. Indian progress alone accounts for almost three quarters of
decline in teen births worldwide. This means now in India more girls manage to stay in school and
have better access to sexual and reproductive health services. This indicates a better socio-
economic progress in the country.

53
www.YouTube.com/SleepyClases
www.SleepyClasses.com
ENVIRONMENT – 3
Questions

1) The IUCN Status of Dugong is:


a) Endangered
b) Critically Endangered
c) Vulnerable
d) None of the above
2) Which of the following correctly defines microplastics?
a) Particles smaller than 5 millimetres in length
b) Particles smaller than 10 millimetres in length
c) Particles smaller than 5 centimetres in length
d) Particles smaller than 5 centimetres in length
3) Consider the following :
1. Solar Energy
2. Wind Energy
3. Bio Energy
4. Small Hydro Energy
Arrange the above in descending order with respect to their installed capacities in the country.
a) 1 > 2 > 3 > 4
b) 2 >1 > 3 > 4
c) 1 > 2 > 4 > 3
d) 2 >1 > 4 > 3
4) The IUCN status of African subspecies of chimpanzee (Pan troglodytes verus) is:-
a) Critically Endangered
b) Endangered
c) Vulnerable
d) Near Threatened
5) Consider the following statements about Ocean deoxygenation or oxygen decline:
1. Hypoxic areas are defined as regions where oxygen below 60 umol/kg.
2. Climate change reduces the ability of ocean to hold oxygen, leading to an oxygen decline.
3. Warmer ocean temperatures increase oxygen demand from organisms further leading to oxygen
decline.
Which of the above statements is/are correct?
a) 1 and 3 only
b) 1 and 2 only
c) 2 and 3 only
d) All of the above
6) Renewable 2019 Global Status Report (GSR) has been released by:
a) International Renewable Energy Agency (IRENA)
b) REN21
c) European Solar Thermal Electricity Association (ESTELA)
d) Renewable Energy Association (REA)

54
www.YouTube.com/SleepyClases
www.SleepyClasses.com
https://youtu.be/6AxfLCQJ1oQ?list=PLYSuG-KZy7JB0ntaozbKLnni_iYiKGOPk

55
www.YouTube.com/SleepyClases
www.SleepyClasses.com
Explanations

1) C
The dugong is a species of marine mammal similar to the American manatee and can grow to
about 3.4 meters (11 feet) in length. Its conservation status is listed as vulnerable by the
International Union for the Conservation of Nature. https://www.thehindu.com/sci-tech/energy-
and-environment/thai-vets-nurturelost-baby-dugong-with-milk-and-grass/article27951038.ece The
dugong is a medium-sized marine mammal. It is one of four living species of the order Sirenia,
which also includes three species of manatees. It is the only living representative of the once-
diverse family Dugongidae; its closest modern relative, Steller's sea cow, was hunted to extinction
in the 18th century. The dugong is the only sirenian in its range, which spans the waters of some
40 countries and territories throughout the Indo-West Pacific. The dugong is largely dependent on
seagrass communities for subsistence and is thus restricted to the coastal habitats which support
seagrass meadows.

56
www.YouTube.com/SleepyClases
www.SleepyClasses.com
2) A
Plastic is a versatile, multi-utility substance used throughout the world. It is susceptible to
disintegration into smaller particles and particles smaller than 5 millimetres are termed
microplastics. Microplastics are omnipresent: they are in the water, soil and even air. However,
there is very little information on the source of different type of microplastics. For instance, one
study has reported that certain cosmetics containing micro-beads is a major source of granular
plastic, while another study has found that a single shirt made of polyester releases approximately
1900 microplastic fibres during every wash. At present, there is no internationally accepted
method for estimation of microplastic content in the soil.
https://www.thehindubusinessline.com/news/science/microplasticpollution-load-on-seashore-
linked-with-coastal-cities-populationsize-study/article27149810.ece
3) B
A total of 74.79 GW of renewable energy capacity has been installed in the country as on
31/12/2018 which includes 25.21 GW from Solar, 35.14 GW from Wind, 9.92 GW from Bio
power, 4.52 GW from Small Hydro Power.

http://pib.nic.in/PressReleseDetail.aspx?PRID=1573837

A target of installing 175 GW of renewable energy capacity by the year 2022 has been set, which
includes 100 GW from solar, 60 GW from wind, 10 GW from bio-power and 5 GW from small
hydro power..
4) A
In 2016, the International Union for Conservation of Nature listed the western African subspecies
of chimpanzee (Pan troglodytes verus) as ―Critically Endangered‖. It had previously been listed as
―Endangered‖. This change of status reflected dramatic declines in population numbers, of 80%
over 24 years. And the outlook for this ape subspecies‘ future is not good. The IUCN predicted
declines of 6 per cent per annum if threats to the chimpanzees weren‘t urgently averted. These
threats are complex. Most of the species lives outside protected areas in West Africa. This means
they are susceptible to extinction because of rapid land conversion that‘s driven especially by
57
www.YouTube.com/SleepyClases
www.SleepyClasses.com
large-scale development projects. Chimpanzees need wild resources for food and nesting; they
mostly eat fruit and typically make a new nest in a tree every night to sleep in. Although
chimpanzees are highly intelligent and adaptable, their ability to survive in environments impacted
by humans depends on several important factors. https://www.downtoearth.org.in/news/wildlife-
biodiversity/action-is-needed-to-save-westafrica-s-critically-endangered-chimpanzees-65031

5) D
Ocean deoxygenation refers to the loss of oxygen from the oceans due to climate change. Long-
term ocean monitoring shows that oxygen concentrations in the ocean have declined during the
20th century, and the new IPCC 5th Assessment Report (AR5 WG1) predicts that they will
decrease by 3-6% during the 21st century in response to surface warming. While 3-6% doesn‘t
seem like much, this decrease will be felt acutely in hypoxic and suboxic areas, where oxygen is
already limiting. ―Hypoxic‖ areas are defined as regions where oxygen limitation is detrimental to
most organisms. This threshold differs across the world, but is usually defined as anything below
60 umol/kg. Hypoxic zones have oxygen concentrations 70-90% lower than the mean surface
concentrations. ―Suboxic‖ areas are areas where oxygen is so low (less than 5 umol/kg) that most
life cannot be sustained and significant biogeochemical changes occur due to altered water
chemistry. Suboxic zones have oxygen concentrations 98% lower than the mean surface
concentrations. A recent study found that a 1°C warming throughout the upper ocean will result in
the increase of hypoxic areas by 10% and a tripling of the volume of suboxic waters. To put this in
context, a highly optimistic emissions scenario of atmospheric CO2 levels of 550 ppm by 2100
would lead to a 1.2°C warming of the upper ocean.
Oxygen content in the water is dependent on photosynthesis (produces oxygen), animal respiration
(uses oxygen), and physical mixing. Ocean warming is reducing global ocean oxygen content
through several key mechanisms including - Stratification impacts: Anthropogenic warming causes
surface waters to become warmer and thereby less dense, leading to a more stratified (layered)
water column, which reduces mixing. Other impacts of climate change to the water cycle can also
lead to a more stratified water column. These include inputs of freshwater to the ocean from rain,
river runoff, or melting ice.

Warming effects: As a physical rule, warmer water holds less oxygen. As the surface waters warm
due to climate change, the ocean loses its ability to hold oxygen, leading to an oxygen decline.

Biological effects: Changes to the biological use and production of oxygen can lead to changes in
oxygen content in the water. Warmer ocean temperatures increase oxygen demand from
organisms. Increased nutrient inputs (either through coastal runoff or through upwelling) also lead
to more oxygen depletion at mid-depths (100-1000m).

Circulation changes: Changes in ocean circulation are also implicated with some of the observed
declines in dissolved oxygen (Grantham et al. 2004). Slowing circulation and increased upwelling
of oxygen-poor deep-water can lead to reductions in oxygen.

https://www.downtoearth.org.in/news/climate-change/largermarine-invertebrates-more-
vulnerable-to-ocean-deoxygenation65114
6) B

58
www.YouTube.com/SleepyClases
www.SleepyClasses.com
Renewable energy‘s share in power consumption is increasing undoubtedly, but people would have
consumed more had policy makers prioritised the sector, according to REN21‘s Renewable 2019
Global Status Report (GSR) released on June 18, 2019. Erratic policy decisions kept the world from
using the sector to its potential in meeting climate change targets, added the report. In India, which
ranked fourth globally for new investment in renewable energy in 2018, the investment decreased 16
per cent compared to 2017, read the report by the think tank that focuses on renewable energy
policy. REN21 is made up of a worldwide community of players from governments,
intergovernmental and non-governmental organisations, industry, science and academia, REN21 is a
global network providing high-quality, up-to-date information to shape the energy debate.

https://www.downtoearth.org.in/news/energy/bad-policy-decisions-in-2018impacted-renewable-
energy-growth-report-65137

59
www.YouTube.com/SleepyClases
www.SleepyClasses.com
ENVIRONMENT – 4
Questions

1) Which of the following best defines a Ghost Net‗?


a) It is an umbrella term describing the portions of the Internet purposefully not open to public view
b) They are the fishing nets that have been abandoned by fishermen or lost in the ocean.
c) They are the fishing nets laced with poisons to kill the marine species
d) It consists of hidden networks whose architecture is superimposed on that of the Internet.
2) Consider the following statements regarding Bonn challenge.
1. It will help to enhance India‗s capacity on forest landscape restoration (FLR).
2. India joined the voluntary Bonn Challenge at the UNFCC Conference of the Parties (COP) 2015
in Paris.
3. It will be implemented at the pan India level for a period of 3.5 years.
Which of the above statements is/are correct?
a) 1 and 2 only
b) 2 and 3 only
c) 1 and 3 only
d) All of the above
3) Consider the following statements regarding recent extinctions.
1. Spix‗s macaws – Brazil
2. White Rhino - Sudan
3. Great Indian Bustard - India
Which of the following species are correctly matched with the locations in which they became
extinct?
a) 2 only
b) 2 and 3 only
c) 1 and 2 only
d) 3 only
4) Which of the following is special about a Bush tomato (Solanum plastisexum) found in the monsoon
tropics of northern Australia?
a) The plant‗s flowers whenever encountered are hermaphrodite, at other times male, and
sometimes a mix of both.
b) It has recently become the first tomato species in the world to become extinct.
c) It had become extinct and was rediscovered recently in Australia.
d) It has found to have medicinal anti cancer properties.
5) Consider the following statements about decommissioning levy.
1. It is collected by Nuclear Power Corp of India Ltd (NCPI), India‗s sole nuclear energy producer.
2. It is managed by the Department of Atomic Energy.
Which of the above statements is/are correct?
a) 1 only
b) 2 only
c) Both of the above
d) None of the above
6) Indian researchers have discovered the world‗s smallest land fern hiding in the Western Ghats. It has
been discovered in which of the following states?
60
www.YouTube.com/SleepyClases
www.SleepyClasses.com
a) Kerala
b) Gujarat
c) Karnataka
d) Maharashtra

https://youtu.be/hl0fWNzBGnk?list=PLYSuG-KZy7JB0ntaozbKLnni_iYiKGOPk

61
www.YouTube.com/SleepyClases
www.SleepyClasses.com
Explanations

1) C
The nets in which the marine mammals get entangled are commonly called ghost nets. Ghost nets
are commercial fishing nets that have been lost, abandoned, or discarded at sea. These nets are
hazardous to turtles and other marine life as they continue to actively catch fish, sea snakes,
cetaceans, turtles and other marine animals. Every year they are responsible for trapping and
killing millions of marine animals including sharks, rays, bony fish, turtles, dolphins, whales,
crustaceans, and birds. Ghost nets cause further damage by entangling live coral, smothering reefs
and introducing parasites and invasive species into reef environments.
https://www.thehindu.com/sci-tech/energy-and-environment/turtles-wrappedin-nets-wash-ashore-
in-goa-saved/article28078272.ece Ghost fishing occurs when lost or discarded fishing gear that is
no longer under a fisherman's control continues to trap and kill fish, crustaceans, marine mammals,
sea turtles, and seabirds. The Olive Ridley Project was founded in 2013 by Biologist Martin
Stelfox in response to the large amounts of entangled olive ridley sea turtles he encountered in the
Maldives.

2) A
Bonn Challenge is a global effort to bring 150 million hectares of the world‗s deforested and
degraded land into restoration by 2020, and 350 million hectares by 2030. At the UNFCC
Conference of the Parties (COP) 2015 in Paris, India also joined the voluntary Bonn Challenge
pledge to bring into restoration 13 million hectares of degraded and deforested land by the year
2020, and additional 8 million hectares by 2030. India‗s pledge is one of the largest in Asia.

http://pib.nic.in/PressReleseDetail.aspx?PRID=1574780
The Union Environment Minister has launched a flagship project on enhancing capacity on forest
landscape restoration (FLR) and Bonn Challenge in India, through a pilot phase of 3.5 years
62
www.YouTube.com/SleepyClases
www.SleepyClasses.com
implemented in the States of Haryana, Madhya Pradesh, Maharashtra, Nagaland and Karnataka.
Ministry of Environment, Forest and Climate Change (MoEFCC) in partnership with The
International Union for Conservation of Nature (IUCN), through this flagship project aims to
develop and adapt best practices and monitoring protocols for the Indian states and build capacity
within the five pilot states on FLR and Bonn Challenge. This will be eventually scaled up across
the country through subsequent phases of the project.
3) C
We‗ve read those stories of recent extinctions Sudan, the last male northern white rhino and
Brazil‗s Spix‗s macaws — with much consternation. But closer home, a tale of extinction may be
unfolding before our very eyes: the great Indian bustard, that narrowly missed being christened
India‗s national bird, is now teetering on its last legs. Several threats — including power lines —
are decimating bustard populations. India, effectively the only home of the bustards, now harbours
less than 150 individuals in five States. (1000 in 11 states in 1969) https://www.thehindu.com/sci-
tech/energy-and-environment/theindian-bustard-on-its-last-legs/article25758411.ece

GREAT INDIAN BUSTARD


The largest population of is found in the state of Rajasthan followed by Gujarat, Madhya Pradesh,
Maharashtra and Andhra Pradesh (BirdLife International, 2015). The GIB‗s last remnant wild
population of about 50 in Jaisalmer district accounts for 95% of its total world population. The
GIB is listed under Schedule I of the Wildlife (Protection) Act, 1972, and listed as an Appendix I
species of CITES. It is the state bird of Rajasthan. Rajasthan has announced its own Project Great
Indian Bustard. In the coming years, they will spend close to Rs 8 crore at the breeding sites of
bustards outside the protected areas. It has also decided to set up two captive breeding centres for
conservation of the Great Indian Bustard. The two places identified for setting up the breeding
centres for the Rajasthan's state bird are Sursan in Kota and Ramdeora in Jaisalmer.
The plan is to breed them at the centre for three-four generations, and then start releasing them into
the wild. The bird has a lifespan of 15-20 years in captivity. The female lays only one egg per
year (two in rare cases), that too only if it deems the ecological conditions fit for the chick‗s
survival. While the bird has a ‗relatively long life, its population growth is slow.

63
www.YouTube.com/SleepyClases
www.SleepyClasses.com
The Union Ministry of Environment and Forests (MoEF) prepared a species recovery programme
for the Great Indian Bustard, the Lesser Florican and the Bengal Florican, three of the four bustard
species found in India. All the three birds have been endemic to the grasslands of India and are on
the brink of extinction. The fourth one, Houbara, is a migratory species.
Threats The GIB prefers grasslands ecosystem to survive (the most threatened and neglected
ecosystem). But, their primary habitat is being diverted for industries, mining, and intensive
agricultural practices. The untamed, arid grasslands that bustards thrive in are categorised as
‗wastelands‗, like most grassland habitats in India. The push to make these areas more
‗productive‗ has seen an increase in water availability in these parts, resulting in the spillover of
agricultural land into bustard habitats. More recently, what remains of their grassland homes are
now sites for renewable power projects. With new wind turbines, come more power lines to take
the ‗green‗ energy to grids and homes. Bustards, with their poor frontal vision and heavy bodies,
cannot manoeuvre away from cables in time. The situation is so dire that three non-profits — the
Corbett Foundation, Conservation India and Sanctuary Nature Foundation — have initiated an
online petition (which has already garnered more than 6,000 signatures) to Union Power Minister
R.K. Singh to demand that power lines be routed underground.
4) A
A team of scientists from the US and Australia has named a new plant species from the northern
Australia as Solanum Plastisexum. First discovered in 1977, it is also known as the Dungowan
bush tomato. (after the cattle station where the species was discovered) For decades, the plant
remained unnamed and no scientists could understand the functionality of the plant. The sex of its
flowers kept changing every time it was studied. The researchers have named the plant Solanum
plastisexum, the second part of which is derived from a Greek root meaning ―moldable‖ or
―changeable,‖ combined with the Latin word for sex. Each time it was encountered, it was
expressing itself a different way through its sexual form. Sometimes the plant‗s flowers were
hermaphrodite, at other times male, and sometimes a mix of both. Australia is among a handful of
countries considered to be megadiverse — meaning it has a vast range of plant and animal life —
but about 70 percent of the country‗s species are undescribed, according to a 2009 government
report. https://www.nytimes.com/2019/06/18/world/australia/tomato-sex-nonbinary.html

64
www.YouTube.com/SleepyClases
www.SleepyClasses.com
5) C
As per the IAEA Safety Standards/Guides, a mechanism for providing adequate financial resources
should be established to cover the costs of radioactive waste management and, in particular, the
cost of decommissioning. It should be put in place before operation and should be updated as
necessary. Consideration should also be given to providing the necessary financial resources in the
event of premature shutdown of a facility. India is yet to start decommissioning nuclear plants
even when there is a guideline to do so by the Atomic Energy Regulatory Board. The Nuclear
Power Corp of India Ltd (NCPI), India‗s sole nuclear energy producer, already collects
―decommissioning levy‖ from users to create a fund to finance decommission activities in the
future. This fund is managed by the Department of Atomic Energy. The 2017-18 annual NCPI
report mentioned that the decommission levy fund has at least Rs 1,975 crore. India has so far
decommissioned research reactors Zerlina and Purnima
https://www.downtoearth.org.in/news/energy/as-renewable-energy-picks-up-
nucleardecommissioning-market-booms-65209
6) B
Indian researchers have discovered the world‗s smallest land fern hiding in the Ahwa forests of the
Western Ghats in Gujarat‗s Dang district. According to a recent study in Scientific Reports, an
international journal that publishes multidisciplinary research, the fingernail-sized fern belongs to a
group known as the adder‗s-tongue ferns, named after their resemblance to a snake‗s tongue. The
size of the new Malvi‗s adder‗s-tongue fern Ophioglossum malviae – just one centimetre – is
probably the reason why it remained hidden all along. A look at the plant‗s minuscule seeds (called
spores) under a powerful electron microscope revealed it had a unique thick outer layer which
similar species lacked. The researchers also analysed the plant‗s DNA and found it to vary enough
from its relatives to call it a new species.

65
www.YouTube.com/SleepyClases
www.SleepyClasses.com
https://www.thehindu.com/sci-tech/science/western-ghats-reveal-worldssmallest-land-
fern/article23661416.ece

66
www.YouTube.com/SleepyClases
www.SleepyClasses.com
SCIENCE & TECH – 1
Questions

1) ‗I am overworked‘ campaign is in the context of which of the following.


a) Corporate Workers
b) Police
c) Doctors
d) Manual Labourers
2) Consider the following statements in the context of Nipah Virus :
1. The first outbreak of Nipah virus in India, took place in Uttarakhand.
2. The symptoms of the virus are similar to those of influenza
a) 1 only
b) 2 only
c) Both 1 and 2
d) Neither 1 nor 2
3) Cataract affects which part of the eye?
a) Lens
b) Pupil
c) Cornea
d) Retina
4) Which of the following may be the reasons for Post-term Pregnancy?
1. It can be hereditary
2. Irregular menstrual cycles
3. Narrow size of pelvis
a) 1 and 2 only
b) 2 and 3 only
c) 1 and 3 only
d) 1,2 and 3
5) Burn-out recently in news is best defined as:
a) a serious health issue by WHO
b) death of people due to PM 2.5
c) death of people due to overwork
d) death due to stubble burning

https://youtu.be/S0vLqWctIXs?list=PLYSuG-KZy7JB0ntaozbKLnni_iYiKGOPk

67
www.YouTube.com/SleepyClases
www.SleepyClasses.com
Explanations

1) C
 Demanding regulated working hours, and standardised working and living conditions, resident
doctors across India have joined hands to start a unique campaign called ―I am overworked‖
 ―As part of the campaign, the doctors will be wearing a band/badge with these words while doing
their duty. There will be no strike or gathering but a silent appeal to the Union Health Minister
 The campaign, meanwhile, is aimed at ―highlighting the issues of unregulated duty hours, poor
working and living conditions and increasing number of depression and suicides among resident
doctors in the country,‖ noted the group of doctors
2) B
 The outbreak of Nipah virus took place in 2018 in Kerala
 The symptoms of Nipah are similar to that of influenza such as fever, muscle pain and
respiratory problems
o It may also present itself in humans in the form of Acute Encephalitis Syndrome (AES)
 But sometimes a person can have an asymptomatic infection and be a carrier of Nipah and not
show any symptoms
3) A
 Inside our eyes we have natural lens which bends (refracts) light rays that come into the eye to
help us see
 If one has cataract, it means the lens has become cloudy
o It is like looking through a foggy or dusty car windshield
 Dr. Patricia Bath, a pioneering ophthalmologist who became the first African American female
doctor to receive a medical patent after she invented a more precise treatment of cataracts, has
died.

4) D

68
www.YouTube.com/SleepyClases
www.SleepyClasses.com
 Usually, the due date is calculated as 280 days or 40 weeks from the past menstrual cycle. The
baby is expected to be full-term after the 38th week
 The reason for post-term pregnancy is not known
o It can be hereditary
o If a woman has a history of post-term pregnancy, then chances of recurrence are high
o It can also happen if the woman has a narrow pelvis, in which case the head of the baby
may not fit in, thus leading to the cervix not opening up
o It is also seen among women with irregular menstrual cycles
5) A
 WHO has classified ‗Burn-out‘ as an occupational phenomenon and not a medical condition
 Burn-out is a syndrome conceptualised as resulting from chronic workplace stress that has not
been successfully managed.
 Characterised by three-dimensions
o feelings of energy depletion or exhaustion
o increased mental distance from one‘s job or feelings of negativism or cynicism related to
one‘s job
 reduced professional efficacy
 It specifically applies to the occupational context and should not be applied to describe
experiences in other areas of life

69
www.YouTube.com/SleepyClases
www.SleepyClasses.com
INTERNATIONAL RELATIONS – 1
Questions

1) Arrange the following cities from East to West :-


1. Ashgabat
2. Tashkent
3. Astana
4. Bishkek
a) 4-2-3-1
b) 1-3-2-4
c) 1-2-3-4
d) 4-3-2-1
2) The territory of Israel shares border with which of the following water bodies:
1. Red Sea
2. Mediterranean Sea
3. Dead Sea
4. Sea of Galilee
a) 1 and 2 only
b) 2 and 3 only
c) 1,2 and 3 only
d) 1,2,3 and 4 only
3) In the context of Generalised System of Preferences (GSP), which of the following statement(s)
is/are true?
1. Only the United States extends the GSP benefits to the developing countries.
2. Only the wholly owned products qualify under the rules of origin.
a) 1 only
b) 2 only
c) Both 1 and 2
d) Neither 1 nor 2
4) Which of the following counties is not a member of Financial Action Task Force (FATF)?
a) Pakistan
b) Russia
c) USA
d) Brazil
5) Which of the following maintains the Currency Monitoring List?
a) Financial Action Task Force (FATF)
b) International Monetary Fund (IMF)
c) World Trade Organisation (WHO)
d) None of the above

https://youtu.be/vATAwWxVUe8?list=PLYSuG-KZy7JB0ntaozbKLnni_iYiKGOPk

70
www.YouTube.com/SleepyClases
www.SleepyClasses.com
Explanations

1) A

2) D

3) D

71
www.YouTube.com/SleepyClases
www.SleepyClasses.com
Generalized System of Preferences (GSP) is a preferential tariff system extended by developed
countries (also known as preference giving countries or donor countries) to developing countries
(also known as preference receiving countries or beneficiary countries).

GSP to Indian goods is presently extended by 29 developed countries. Only such products of a
beneficiary country (like India) that fulfil the requirements of the rules of origin laid down by the
importing country, are considered eligible for preferential tariff treatment on import into the markets
of donor countries. Rules of origin comprise a set of requirements laid down by the importing
country, which must be fulfilled by a product to be eligible for preferential tariff treatment upon
import in that country.
Products exported from India can be divided into two groups — Wholly Obtained Products and
Products with Import Content. Wholly Obtained Products are those, which have been entirely
 Grown
 Extracted from the Soil
 Harvested within the country
 Manufactured exclusively from the above
Products With Import Content qualify for GSP if the materials, parts or components of imported
or unknown origin used in their manufacture have undergone Sufficient Working or Processing
in India. On the other hand, a product using imported materials is non-originating if the imported
materials have only been subject to minimal operations in India
4) A

5) D
 The Trump administration on Tuesday removed India from its Department of Treasury‘s
currency monitoring list of major trading partners, citing certain developments and steps being
taken by New Delhi which address some of its major concerns. Switzerland is the other nation
that has been removed by the US from its currency monitoring list which among others include
China, Japan, South Korea, Germany, Italy, Ireland, Singapore, Malaysia and Vietnam.

72
www.YouTube.com/SleepyClases
www.SleepyClasses.com
India for the first time was placed by the US in its currency monitoring list of countries with
potentially questionable foreign exchange policies in May 2018 along with five other countries -
China, Germany, Japan, South Korea and Switzerland.

73
www.YouTube.com/SleepyClases
www.SleepyClasses.com
INTERNATIONAL RELATIONS – 2
Questions

1) ‗Windrush Scheme‘ for citizenship has been proposed by which of the following countries?
a) Saudi Arabia
b) United Arab Emirates
c) United Kingdom
d) United States of America
2) Which of the following statement(s) is/are correct in the context of Global Peace Index (GPI)?
1. India‘s ranking has improved significantly over the previous 5 years.
2. Afghanistan is declared as the least peaceful country.
a) 1 only
b) 2 only
c) Both 1 and 2
d) Neither 1 nor 2
3) In the context of G20 Summit, which of the following statement(s) is/are true?
1. India is going to host the G20 summit in 2020.
2. G20 economies account for nearly 90% of the gross world product and two thirds of the world
population.
a) 1 only
b) 2 only
c) Both 1 and 2
d) Neither 1 nor 2
4) Kimberley Process is related to which of the following?
a) Peace in Afghanistan
b) Conflict diamonds
c) Illegal trade of endangered animals
d) Climate Change
5) In the context of CICA, which of the following statement(s) is/are true?
1. India has been one of the founding member of the conference.
2. The decisions in CICA are taken by the majority vote.
a) 1 only
b) 2 only
c) Both 1 and 2
d) Neither 1 nor 2

https://youtu.be/POl0SIis0JI?list=PLYSuG-KZy7JB0ntaozbKLnni_iYiKGOPk

74
www.YouTube.com/SleepyClases
www.SleepyClasses.com
Explanations

1) C
 Windrush Generation refers to citizens of former British colonies who arrived in the UK before
1973, when the rights of such Commonwealth citizens to live and work in Britain were
substantially curtailed

 Indians emerged as one of the largest groups affected, after Caribbean nationals, in the scandal
involving Commonwealth nationals wrongly denied their citizenship rights in Britain

 The immigrants referred to under the bracket of 'Windrush Generation' relates to a ship named
'Windrush', which brought Jamaican workers to the UK shores in 1948

 Windrush Scheme ensures that members of this generation, their children born in UK and those
who arrived in the UK as minors will be able to apply for citizenship, or other immigration
products free of charge
2) B
 India has slipped by four points in annual global index on peacefulness, finishing at 141 among
163 countries, released by the Institute for Economics and Peace (IEP)

 GPI presents the most comprehensive data driven analysis to date on peace, its economic value,
trends, and how to develop peaceful societies

 India‘s rank has moved down to 141 in 2019 from 136 in 2018

 In 2019 report, the average level of global peacefulness improved for the first time in five years.
However, despite improvement, the world remains considerably less peaceful now than a decade
ago

o The report also includes new research on the possible effects of climate change on peace
 Afghanistan is now the least peaceful country, replacing Syria
3) B
 G-20 is a grouping of the world‘s 20 major economies. Collectively, they account for nearly 90%
of the gross world product, 80% of world trade, twothirds of the world population, and
approximately half of the world land area.
 India will host the G-20 summit in 2022. Italy was to host the international forum in 2022, but
since the year marks the 75th anniversary of India‘s Independence, so Italy allowed India to play
the host
 G-20 members comprise Argentina, Australia, Brazil, Canada, China, the European Union,
France, Germany, India, Indonesia, Italy, Japan, Mexico, Russia, Saudi Arabia, South Africa,
South Korea, Turkey, the UK and the U.S.
 Spain is a permanent guest invitee
4) B
75
www.YouTube.com/SleepyClases
www.SleepyClasses.com
 The Kimberley Process (KP) unites administrations, civil societies, and industry in reducing the
flow of conflict diamonds - ‗rough diamonds used to finance wars against governments‘ - around
the world
 India holds currently the chair of KP, which oversees the implementation of KPCS
 KP members are responsible for stemming 99.8% of the global production of conflict diamonds
5) A

76
www.YouTube.com/SleepyClases
www.SleepyClasses.com
SCI & TECH – 2
Questions

1) Pink Bollworm is related to which of the following crops?


a) Rice
b) Wheat
c) Cotton
d) Pulses
2) Which of the following thing would an astronaut observe when in the Space Station?
1. Candle burns with the same tear-shaped flame
2. Water does not bubble when it boils
a) 1 only
b) 2 only
c) Both 1 and 2
d) Neither 1 nor 2
3) Which of the following are the by-products of a fuel cell?
a) Water
b) Carbon dioxide
c) Hydrogen Sulphide
d) Methane
4) Which of the following statement(s) in the context of Anthrax are true?
1. Anthrax is a viral disease and can also infect animals like horses, sheep, cattle and goats.
2. It is a contagious disease and can spread through droplet infection
a) 1 only
b) 2 only
c) Both 1 and 2
d) Neither 1 nor 2
5) Which of the following statement(s) is are true in the context of Genetically modified crops?
1. Bt brinjal has been approved for seed production by the GEAC.
2.It is the first GM food crop that has been cultivated in India.
a) 1 only
b) 2 only
c) Both 1 and 2
d) Neither 1 nor 2

https://youtu.be/0bsFYFinkKo?list=PLYSuG-KZy7JB0ntaozbKLnni_iYiKGOPk

77
www.YouTube.com/SleepyClases
www.SleepyClasses.com
Explanations

1) C
2) B
3) A
 A fuel cell uses hydrogen has (H2) and oxygen gas (O2) as fuel
 The products of the reaction in the cell are water, electricity and heat
 A team of scientists from India has developed a selenium-graphene-based catalyst which is more
efficient, costs less and also remains stable for longer than the usual platinum based catalysts
4) D
 Anthrax is a serious infection disease caused by rod-shaped bacteria known as Bacillus anthracis
 It can be found naturally in soil and commonly affects domestic and wild animals around the
world
 People can get sick with anthrax if they come in contact with infected animals or contaminated
animal products
 Humans, pigs and dogs are comparatively less susceptible and only get infected if exposed to
copious amount of spores.
 It is not contagious, which means it cannot spread through cold or flu
5) A
 GEAC has approved large-scale field trails and seed production, but the clearance is facing stiff
resistance from farmers, environmentalists and consumer groups
 Bt cotton is the only other gm crop permitted in the country.
 Of the three gm food crops in India -- brinjal, mustard and potato -- awaiting permission for
large-scale field trials and seed production, brinjal is the first one to get geac approval.

78
www.YouTube.com/SleepyClases
www.SleepyClasses.com
GEOGRAPHY – 1
Questions

1) Which of the following is true in context with Quinary activities?


1. Quinary activities are services that focus on the creation, re-arrangement and interpretation of
new and existing ideas; data interpretation and the use and evaluation of new technologies.
2. These are often referred to as White Collar Professionals.
3. They represent another subdivision of the tertiary sector representing special and highly paid
skills of senior business executives, government officials, research scientists, financial and legal
consultants, etc.
a) 1 and 2 only
b) 2 and 3 only
c) 1 and 3 only
d) 1,2 and 3
2) Which of the following best describes the word ―Possibilism‖ in context with ―Human
Geography‖?
a) The Environment sets certain constraints or limitations, but culture is otherwise determined by
social conditions.
b) It explains how the physical environment predisposes societies and states towards particular
development trajectories.
c) It explores the new-found possibilities of Origin of Universe.
d) None of the above
3) ―Rutland Island ―was in news recently. It lies in :
a) Bay of Bengal
b) Arabian Sea
c) Indian Ocean
d) None of the above
4) Which of the following crops are cultivated under ―dryland farming‖?
1. Sugarcane
2. Jute
3. Guar
a) 1 only
b) 2 only
c) 3 only
d) All of the above
5) Match the following
Volcano : Place
1. Mount Vesuvius : (a) Hawaii
2. Mauna Loa : (b) Italy
3. Cotopaxi : (c) Equador
4. Mount Pinatubo : (d) Philippines
a) 1-a; 2-b; 3-c; 4-d
b) 1-b; 2-c; 3-a;4-d
c) 1-b; 2-a; 3-c;4-d
d) 1-a; 2-c; 3-d;4-b
79
www.YouTube.com/SleepyClases
www.SleepyClasses.com
https://youtu.be/1ty1p5K8MVs?list=PLYSuG-KZy7JB0ntaozbKLnni_iYiKGOPk

80
www.YouTube.com/SleepyClases
www.SleepyClasses.com
Explanations

1) C
Quinary activities are services that focus on the creation, re-arrangement and interpretation of new
and existing ideas; data interpretation and the use and evaluation of new technologies.

Often referred to as ‗gold collar‘ professions, they represent another subdivision of the tertiary
sector representing special and highly paid skills of senior business executives, government
officials, research scientists, financial and legal consultants, etc.
2) A
Possibilism in Human geography is the theory that the environment sets certain constraints or
limitations, but culture is otherwise determined by social conditions.
3) A
RUTLAND ISLAND
• It is in South Andaman
• It will be developed as DRDO‘s long range missile testing facility
• Island is very rich in Marine life
• The project was continuously stalled by the environment ministry as the project involves
diversion of 49.978 acres forest land, of which 0.84 ha falls in the Mahatma Gandhi Marine
National Park and 49.138 ha reserve forest within 10km of the Eco Sensitive Zone.
• National Board of Wildlife as given its approval now due to strategic importance of the project
4) C
In India, the dryland farming is largely confined to the regions having annual rainfall less than 75
cm. These regions grow hardy and drought resistant crops such as ragi, bajra, moong, gram and
guar (fodder crops) and practise various measures of soil moisture conservation and rain water
harvesting. In wetland farming, the rainfall is in excess of soil moisture requirement of plants
during rainy season. Such regions may face flood and soil erosion hazards. These areas grow
various water intensive crops such as rice, jute
5) C
• Mount Vesuvius : (a) Italy
• Mauna Loa : (b) Hawaii
• Cotopaxi : (c) Equador
• Mount Pinatubo : (d) Philippines

81
www.YouTube.com/SleepyClases
www.SleepyClasses.com
GEOGRAPHY – 2
Questions

1) ‗Push and Pull factors‘ are two sets of factors that influence ‗Migration‘ . Which of the following
are true in context with the same?
1. Pull factors make the place of origin seem less attractive for reasons like unemployment, poor
living conditions, political turmoil, unpleasant climate, natural disasters, epidemics and socio-
economic backwardness.
2. Push factors make the place of destination seem more attractive than the place of origin for
reasons like better job opportunities and living conditions, peace and stability, security of life and
property and pleasant climate.
a) 1 only
b) 2 only
c) Both 1 and 2
d) Neither 1 nor 2
2) The term ―Urban Sprawl‖ is often in news . Which one among the following best describes the
term.
a) A medium-sized human settlement that is generally larger than a village but smaller than a city,
b) An increase in a population in cities and towns versus rural areas.
c) It refers to the migration of a population from populated towns and cities to low density
residential development over more and more rural land.
d) None of the above
3) Which of the following straits are not associated with String of Pearls?
1. Strait of Malacca
2. Bering Strait
3. Lombok Strait
a) 1 only
b) 2 only
c) 3 only
d) None of the above
4) Akademik Lomonosov has been in news recently. It is :
a) World‘s Only Floating Nuclear Power Unit
b) World‘s Only Floating Wetland
c) World‘s Only Mach-5 Spacecraft
d) None of the above
5) Which of the following is incorrect in context with ‗Truck Farming‘?
1. The regions where farmers specialise in fruits only, the farming is known as truck farming.
2. The distance of truck farms from the market is governed by the distance that a truck can cover
overnight, hence the name truck farming.
a) 1 only
b) 2 only
c) Both 1 and 2
d) Neither 1 nor 2
https://youtu.be/Dn8E6CuTBi0?list=PLYSuG-KZy7JB0ntaozbKLnni_iYiKGOPk

82
www.YouTube.com/SleepyClases
www.SleepyClasses.com
Explanations
1) D
Migration may be permanent, temporary or seasonal. It may take place from rural to rural areas,
rural to urban areas, urban to urban areas and urban to rural areas. People migrate for a better
economic and social life. There are two sets of factors that influence migration. The Push factors
make the place of origin seem less attractive for reasons like unemployment, poor living
conditions, political turmoil, unpleasant climate, natural disasters, epidemics and socio-economic
backwardness. The Pull factors make the place of destination seem more attractive than the place
of origin for reasons like better job opportunitiesand living conditions, peace and stability, security
of life and property and pleasant climate.
2) C
Urban Sprawl
It refers to the migration of a population from populated towns and cities to low density residential
development over more and more rural land. The end result is the spreading of a city and its
suburbs over more and more rural land. In other words, urban sprawl is defined as low density
residential and commercial development on undeveloped land. Most of the time, people will move
from these areas to try to find better areas to live.
3) B
The String of Pearls is a geopolitical theory on potential Chinese intentions in the Indian Ocean
region (IOR). It refers to the network of Chinese military and commercial facilities and
relationships along its sea lines of communication, which extend from the Chinese mainland to
Port Sudan in the Horn of Africa. The sea lines run through several major maritime choke points
such as the Strait of Mandeb, the Strait of Malacca, the Strait of Hormuz, and the Lombok Strait as
well as other strategic maritime centers in Pakistan, Sri Lanka, Bangladesh, the Maldives, and
Somalia.

4) A
It is the world‘s only floating nuclear power unit. (Russia)

83
www.YouTube.com/SleepyClases
www.SleepyClasses.com
5) A
• The regions where farmers specialise in vegetables only, the farming is know as truck farming. •
The distance of truck farms from the market is governed by the distance that a truck can cover
overnight, hence the name truck farming

84
www.YouTube.com/SleepyClases
www.SleepyClasses.com
GEOGRAPHY – 3
Questions

1) Which of the following is matched incorrectly?


1. GOLDEN TRIANGLE : Myanmar, Laos, and Thailand
2. GOLDEN CRESCENT: Afghanistan, Iran, and Pakistan
a) 1 only
b) 2 only
c) Both 1 and 2
d) Neither 1 nor 2
2) Which of the following are true with regard to Krem Puri?
1. It is the longest limestone cave in the world.
2. It is located in the state of Meghalaya.
3. It is located in the district of Mawsynram which is the wettest place on earth
4. Krem Puri cave system also has fossils of dinosaurs, especially Mosasaurus, a giant reptile that
lived 66-76 million years ago
a) 1,2 and 3 only
b) 2,3 and 4 only
c) None of the above
d) All of these
3) Kalahari Desert lies in :
1. Angola
2. Namibia
3. South Africa
4. Botswana
5. Zambia
a) 1,4 and 5 only
b) 1,2,3 and 4 only
c) 2,3 and 4 only
d) 1,2 and 3 only
4) Consider the following statements:-
- This layer of earth‘s crust makes up about 85% of the volume of the earth
- Its density varies between 3.4-3.5 g/cubic cm to 5.5 g/ cubic cm
- It contains dense, coarse grained igneous rocks mostly made of minerals like olivine and
pyroxene
Which layer of earth is explained here?
a) Crust
b) Mantle
c) Outer core
d) Inner core
5) Match the following:-
a. San or Bushman tribe i. Sahara desert
b. Tuaregs ii. Central Asia
c. Tatars iii. Kalahari desert
a) a-i, b-ii, c-iii
85
www.YouTube.com/SleepyClases
www.SleepyClasses.com
b) a-iii, b-ii, c-i
c) a-iii, b-i, c-ii
d) a-ii, b-i, c-iii

https://youtu.be/kiW0VJq7rBw?list=PLYSuG-KZy7JB0ntaozbKLnni_iYiKGOPk

86
www.YouTube.com/SleepyClases
www.SleepyClasses.com
Explanations
1) D
GOLDEN TRIANGLE

• It is the region between the borders of Myanmar, Laos, and Thailand; a famous region for its
opium production.

GOLDEN CRESCENT:
• It is the second major area of illicit opium production in Asia spanning across three nations;
Afghanistan, Iran, and Pakistan. It is located at the crossroads of Central, South and Western Asia.
2) B
The world‘s longest sandstone cave named Krem Puri was discovered near Laitsohum village in
Mawsynram area in East Khasi Hills district of Meghalaya. The cave is 24,583 metres (24.5 km) in
length and is known for its complex cave systems hidden under its undulating hills. Krem Puri
underground cavern is more than 6,000 metres longer than world record-holder Cueva Del Saman
in Edo Zulia, Venezuela, a quartzite sandstone cave measuring 18,200 metres (18.2km). This
sandstone cave is also India‘s second longest cave in general category after limestone Krem Liat
Prah-Umim-Labit system measuring little over 31 km in Jaintia Hills, Meghalaya. Krem Puri cave
system also has fossils of dinosaurs, especially Mosasaurus, a giant reptile that lived 66-76 million
years ago.
3) C

4) B
87
www.YouTube.com/SleepyClases
www.SleepyClasses.com
The mantle extends from Moho‘s discontinuity (35 km) to a depth of 2,900 km (Moho-
Discontinuity to the outer core).

The crust and the uppermost part of the mantle are called lithosphere. Its thickness ranges from 10-
200 km. The lower mantle extends beyond the asthenosphere. It is in solid state. The density of
mantle varies between 2.9 and 3.3. The density ranges from 3.3 to 5.7 in the lower part. It is
composed of solid rock and magma. It forms 83 per cent of the earth‘s volume. The outer layer of
the mantle is partly simatic while the inner layer is composed of wholly simatic ultra-basic rocks.
5) C
a.San or Bushman tribe iii. Kalahari desert
b. Tuaregs i. Sahara desert
c. Tatars ii. Central Asia

88
www.YouTube.com/SleepyClases
www.SleepyClasses.com
GEOGRAPHY – 4
Questions

1) Tamil Nadu coast remains dry during the summer monsoon season? Identify the reason for the
same?
1. Tamil Nadu coast is situated parallel to the Bay of Bengal branch of southwest monsoon.
2. It lies in the rain shadow area of the Arabian Sea branch of the southwest monsoon.
a) 1 only
b) 2 only
c) Both 1 and 2
d) Neither 1 nor 2
2) Which of the following states share its borders with Bangladesh?
1. Mizoram
2. Tripura
3. Nagaland
Choose the correct option:-
a) 1 and 2 only
b) 2 and 3 only
c) 1 and 3 only
d) All of the above
3) In which of the following regions of India are shale gas resources found? (UPSC 2016).
1. Cambay Basin
2. Cauvery Basin
3. Krishna-Godavari Basin
Select the correct answer using the code given below.
a) 1 and 2 only
b) 3 only
c) 2 and 3 only
d) 1,2 and 3 only
4) Oxygen gas is in negligible quantity at the height of atmosphere:
a) 90 km
b) 100 km
c) 120 km
d) 150 km
5) Which one of the following Union Territories of India has the highest literacy rate as per 2011
Census?
a) Lakshadweep
b) Chandigarh
c) Daman and Diu
d) Andaman and Nicobar Islands

https://youtu.be/30pTpf3waws?list=PLYSuG-KZy7JB0ntaozbKLnni_iYiKGOPk

89
www.YouTube.com/SleepyClases
www.SleepyClasses.com
Explanations
1) C

Tamil Nadu coast remains dry during this season. There are two factors responsible for it: (i)
The Tamil Nadu coast is situated parallel to the Bay of Bengal branch of southwest monsoon. (ii)
It lies in the rain shadow area of the Arabian Sea branch of the southwest monsoon.
2) A

90
www.YouTube.com/SleepyClases
www.SleepyClasses.com
3) D
India has identified six basins as areas for shale gas exploration:
1) Cambay (Gujarat),
2) Assam-Arakan (North East),
3) Gondwana (Central India),
4) Krishna Godavari onshore (East Coast),
5) Cauvery onshore,
6) Indo-Gangetic basins.
4) C
The atmosphere is composed of gases, water vapour and dust particles. The proportion of gases
changes in the higher layers of the atmosphere in such a way that oxygen will be almost in
negligible quantity at the height of 120 km. Similarly, carbon dioxide and water vapour are found
only up to 90 km from the surface of the earth.
5) A
Lakshadweep has the highest literacy rate. Refer to:
https://en.wikipedia.org/wiki/List_of_Indian_states_and_union_terr_itories_by_literacy_rate

91
www.YouTube.com/SleepyClases
www.SleepyClasses.com

Anda mungkin juga menyukai